Preclinical Biochemistry and Medical Genetics Review 2023: For USMLE Step 1 and COMLEX-USA Level 1 (USMLE Prep)

The only official Kaplan Preclinical Biochemistry and Medical Genetics 2023 covers the comprehensive information you nee

289 114 449MB

English Pages 409 Year 2023

Report DMCA / Copyright

DOWNLOAD FILE

Polecaj historie

Preclinical Biochemistry and Medical Genetics Review 2023: For USMLE Step 1 and COMLEX-USA Level 1 (USMLE Prep)

Table of contents :
Title
Copyright
Editors
Table of Contents
PART I: BIOCHEMISTRY
CHAPTER 1: Nucleic Acid Structure and Organization
CHAPTER 2: DNA Replication and Repair
CHAPTER 3: Transcription and RNA Processing
CHAPTER 4: The Genetic Code, Mutations, and Translation
CHAPTER 5: Regulation of Eukaryotic Gene Expression
CHAPTER 6: Genetic Strategies in Therapeutics
CHAPTER 7: Techniques of Genetic Analysis
CHAPTER 8: Amino Acids, Proteins, and Enzymes
CHAPTER 9: Hormones
CHAPTER 10: Vitamins
CHAPTER 11: Energy Metabolism
CHAPTER 12: Glycolysis and Pyruvate Dehydrogenase
CHAPTER 13: Citric Acid Cycle and Oxidative Phosphorylation
CHAPTER 14: Glycogen, Gluconeogenesis, and the Hexose Monophosphate Shunt
CHAPTER 15: Lipid Synthesis and Storage
CHAPTER 16: Lipid Mobilization and Catabolism
CHAPTER 17: Amino Acid Metabolism
CHAPTER 18: Purine and Pyrimidine Metabolism
PART II: MEDICAL GENETICS
CHAPTER 1: Single-Gene Disorders
CHAPTER 2: Population Genetics
CHAPTER 3: Cytogenetics
CHAPTER 4: Recombination Frequency
CHAPTER 5: Genetic Diagnosis
INDEX

Citation preview

PRECLINICAL

BIOCHEMISTRY

AND

MEDICAL

GENETICS

REVIEW

2023

® FOR

USMLE

STEP

1

AND

® COMLEX

USA

USMLE®

and

United

(FSMB),

which

are

Board

of Osteopathic

States not

Medical

affiliated Medical

with

LEVEL

Licensing Kaplan

Examiners,

Examination® and

Inc.

were

not

(NBOME),

1

is a joint involved which

program

in the

of the

production

is not affiliated

with

National of, and Kaplan

Board

of Medical

do not endorse, and

was

not

this involved

Examiners product. in the

(NBME) COMLEX production

and

the

Federation

of State

USA® is a registered of, and

does

not

Medical

trademark

endorse,

this

Boards

of the product

National

PRECLINICAL

BIOCHEMISTRY

AND

MEDICAL

GENETICS

REVIEW

2023

FOR

USMLE®

STEP

COMLEX

USA®

USMLE® and of State COMLEX was

United

Medical

States

Boards

Medical

(FSMB),

USA® is a registered

not involved

in the production

Licensing

Examination®

are not affiliated

trademark

of the National does

AND

LEVEL

which of, and

1

is a joint program

with Kaplan Board

not endorse,

1

and

were

of Osteopathic

this product.

of the National not involved Medical

Board

of Medical

in the production

Examiners,

Examiners

of, and

Inc. (NBOME),

(NBME)

do not endorse, which

and the Federation this product.

is not affiliated

with Kaplan

an

USMLE®

and

(NBME) the

United

and

the

not

of,

USA®

This

with

publication

date,

with

medical,

the

the

delivery

of

1515

by

10

7

Creek

ISBN:

Course

Kit

Retail

ISBN:

Retail

Kit

items

Kaplan

978

1

ISBN:

978

come

as

Publishing

the

2

1

5062 1

866

are

not

of

the

National

affiliated

with

Board

Kaplan

of

and

Medical

were

Examiners

not

involved

(NBOME),

which

in

of

production

best

extent

practice

sought.

of

the

and

or

The

any

product.

covered

other

of

is

for

the

as not

expert

intended nor

use

this

publisher

or

not

Publisher to

Inc.

matter

advice is

Examiners,

endorse,

subject

evolve. legal

the

related

not

the

publication

neither

of

does

to

or

This

law,

out

Medical

constantly

If medical

be

arising

of,

in regard

service.

property

Osteopathic

material

reproduced

in

its

use

in

assume

publication

engaged

in

assistance

Editors

the

of

is

rendering

is required,

clinical

practice

any

liability

contained

in this

manner

whatsoever

or for

any

book.

LLC

this

of

Kaplan

publication,

North

America,

LLC.

any

thereof,

may

part

not

be

any

without

1

8445 5062

3 8465

8443 5062

8463

print

books

are

For

more

1949

1

9

and

506

or

publisher.

a set

purposes. at

of

5062

978

and

fullest

Board

information

should

a division

text

3

1

978

educational

department

4

accurate

or

which

program

Road

from

ISBN:

To the

joint

33309

The

5

National

professional

persons

(FSMB),

in the

knowledge

America,

Florida

6

provide

other

Medical,

reserved.

Course

or

Kaplan

Cypress

8

care.

the

involved

professional

to

Boards

is a

product.

of

not

that or

North

permission

9

was

to

competent

Lauderdale,

written

Kit

a

Kaplan

by

All rights

and

Examination®

this

trademark

Kaplan

damage

West

Fort

Medical

State endorse,

registered

medical

and/or

Published

of not

accounting, of

2023

do

understanding

legal,

services

©

Licensing

is designed

the

injury

and

is a

affiliated

Medical

Federation

production

COMLEX

States

should

7

not

be

available information

broken

at

out

special or

to

and

quantity purchase

sold

separately.

discounts books,

to please

use call

for the

sales Simon

promotions, & Schuster

employee special

premiums, sales

Editor

Sam Professor,

Turco,

PhD

Department

University

of

of

Kentucky

Biochemistry

College

Lexington,

of

Medicine

KY

CONTRIBUTORS

Roger Professor, University

Lane,

PhD

Department of

South

of

Alabama

College

Mobile,

Ryan

M.

MD,

Family Family

Professor, University

Department of

Minnesota

of

Health

MS

Clinic,

Ltd

MN

Family

Medicine

Medical Duluth,

Medicine

Medicine

Sandstone,

Assistant

of

AL

Harden,

Physician, Gateway

Biochemistry

School, M

and

Community

Duluth

Campus

Health

We

want

to

hear

what Please

you email

think. us

What at

do

you

like

[email protected]

or

not

like

about

the

Notes?

Table

PART

I:

of

Contents

BIOCHEMISTRY

CHAPTER

1:

Nucleic

Acid

CHAPTER

2:

DNA

CHAPTER

3:

Transcription

CHAPTER

4:

The

CHAPTER

5:

Regulation

CHAPTER

6:

Genetic

CHAPTER

7:

Techniques

CHAPTER

8:

Amino

CHAPTER

9:

Hormones

CHAPTER

10:

Vitamins

CHAPTER

11:

Energy

CHAPTER

12:

Glycolysis

CHAPTER

13:

Citric

CHAPTER

14:

Glycogen,

Structure

Replication

and

and

and

Genetic

of

and

.

.

.

.

.

.

.

.

.

.

.

.

.

.

.

.

.

.

17

.

.

.

.

31

.

.

.

.

.

.

.

.

.

.

.

.

.

.

.

.

.

.

.

.

.

.

.

.

.

.

.

.

.

.

.

.

.

47

Translation

Expression

Analysis

Proteins,

.

.

Therapeutics

Genetic

.

.

and

Gene

in

.

.

Enzymes

.

.

.

.

.

.

.

.

.

.

.

.

.

71

.

.

.

.

.

.

.

.

.

.

.

.

.

.

.

.

83

.

.

.

.

.

.

.

.

.

.

.

.

.

.

.

.

99

.

.

.

.

.

.

.

.

.

.

.

.

.

.

.

115

.

.

.

.

.

.

.

.

.

.

.

.

.

.

.

131

.

.

.

.

.

.

.

.

.

.

.

.

.

.

.

.

.

.

.

.

.

.

.

.

.

.

.

.

.

.

.

.

.

.

.

145

.

.

.

.

.

.

.

.

.

.

.

.

.

.

.

.

.

.

.

.

.

.

159

.

.

.

.

.

.

.

.

.

.

.

.

171

.

.

.

.

.

.

.

.

.

189

and

Cycle

Pyruvate

Dehydrogenase

and

Oxidative

Phosphorylation

Gluconeogenesis,

Monophosphate

and

Shunt

CHAPTER

15:

Lipid

Synthesis

and

CHAPTER

16:

Lipid

Mobilization

CHAPTER

17:

Amino

Acid

Metabolism

CHAPTER

18:

Purine

and

Pyrimidine

.

.

.

Metabolism

Acid

.

Processing

Eukaryotic

of

.

Mutations,

Strategies

Acids,

Repair

RNA

Code,

Organization

.

.

Storage

and

the

.

.

.

.

.

.

Catabolism

.

.

.

Metabolism

.

.

Hexose .

.

.

.

.

.

.

.

.

.

.

.

.

.

.

201

.

.

.

.

.

.

.

.

.

.

.

.

.

.

.

217

.

.

.

.

.

.

.

.

.

.

.

.

.

.

.

237

.

.

.

.

.

.

.

.

.

.

.

.

.

.

.

259

.

.

.

.

.

.

.

.

.

.

.

.

.

.

.

283

3

PART

INDEX

v

II:

MEDICAL

GENETICS

CHAPTER

1:

Single

CHAPTER

2:

Population

CHAPTER

3:

Cytogenetics

CHAPTER

4:

Recombination

CHAPTER

5:

Genetic

.

.

.

.

.

.

.

Gene

Disorders

Genetics

.

.

.

.

.

.

.

.

.

.

.

.

.

.

.

.

.

.

.

.

.

.

.

. 297

.

.

.

.

.

.

.

.

.

.

.

.

.

.

.

.

.

.

.

.

.

. 325

.

.

.

.

.

.

.

.

.

.

.

.

.

.

.

.

.

.

.

.

.

. 339

.

.

.

.

.

.

.

.

.

.

.

.

.

.

.

.

.

. 357

.

.

.

.

.

.

.

.

.

.

.

.

.

.

.

.

.

.

. 369

.

.

.

.

.

.

.

.

.

385

Frequency

Diagnosis

.

.

.

.

.

.

.

.

.

.

.

.

.

.

.

.

.

.

.

.

.

.

PART

BIOCHEMISTR

I

Nucleic

Acid 1

Structure

LEARNING



Explain



Use



Understand

information

of

be

all

the

are

to

able

of by

of

to

life.

DNA

versus

store

The

preserve

its

and

major

nomenclature

RNA

BIOLOGY

and

central

and

a chromosome

generations,

the

structure

MOLECULAR

future

processes

illustrated

nucleotide

of

features

OF

must along

to

organization

general

DOGMA

organism

tion

related

knowledge

information out

Organization

OBJECTIVES

CENTRAL

An

and

steps

dogma

genetic

express involved

of

information,

that in

molecular

pass

information

as

handling

genetic

it

that carries

informa

biology.

Replication Transcription

Translation

DNA

RNA

Protein

Reverse transcription

Figure

Genetic

FigureI 1 I 1. 1 1.Central Central

information

during

the

proteins

is stored

process

made

of by

an

gene

in

the

Dogma Dogma

base

expression,

of MolecularBiology of Molecular

sequence this

of

DNA

information

Biology

molecules.

is

used

Ultimately,

to

synthesize

all

the

organism. HY

HY Classically,

a gene

molecule. tion

is

Although

of

the

ways

a unit this

in

of

the

DNA

definition

which

genes

is may

that now be

encodes

a

particular

complicated

by

expressed,

it is

protein

our

still

or

increased

useful

RNA

apprecia MY

as

LY

definition.

Gene

Expression

Gene

expression

stage

in

double molecule.

and and

gene

If

the

RNA

of

replication

are transfer

molecule molecule the

High

Replication

involves

DNA

converts

DNA

DNA

expression,

stranded

translation sequence

MY

a working

information

to is

the

compared of

base

sequence

a messenger in

the

of

RNA, RNA

found

base

then

a single the

sequence

in

YIELDthe

first

YIELD

MEDIUM

YIELD

a

LOW stranded process

HIGH

Yield

MEDIUM Transcription,

below.

information

LY

YIELD RNA

known

to the amino FUNDAMENTALS

LOW

YIELD

as acid FUNDAMENTALS

a protein. REINFORCEMENT

REINFORCEMENT

3

Immunology

PART

I

|

BIOCHEMISTRY

Biochemistry When

cells

genetic

divide,

duplicated

Medical Genetics

each

information.

Table

before

I 1

Gene

1.

daughter

DNA cell

cell

must

replication

is

receive the

of

Gene

Expression

and

Expression

DNA

DNA

all

organism

requires

the

accurate in

copy

which

of

each

the

chromosome

is

division.

Comparison

Produces

an

process

proteins

an

Replication

Replication

Duplicates

the

before

cell

chromosomes

division

Behavioral Science/SocialSciences Transcription a

of

small

DNA:

section

of

(average

size

of

104–105

nucleotide

Transcription

human

gene,

occurs

Translation

of

RNA

sis)

in

the

throughout

the

concept

events

in

divides

of a

to

divisions

or

Interphase



be •

in

a

tion

entire

chromosome

of

108

Occurs

synthe

Replication

1)

is

human

chromo

nucleotide

during

S

in

pairs)

phase

nucleus

(DNA

of

called

the

is

the

timing

the

time

describes occurs

the

throughout

cellular

cycling,

of in

time all

some

of

which

the

between stages

these cell

2 cell of

interphase.

is

end

growth

such

as

preceding

muscle

and

time

during

DNA nerve

synthesis.

cells,

are

said

G0.

synthesis) At

is

follows:

a period

state

describe

(mitosis)

Interphase

stopped

special

to

phase

expression as

have

used

M

cells.

Gene

(gap

be

The

daughter

and

nucleus

can

cell.

occurs.

content

some,

the

cycle

subdivided

S phase

of size

cycle

cell

2

that

copy

(average

(protein

cell

the

phase

Cells

DNA

cytoplasm

mitoses. is

G1

in

eukaryotic form

of

pairs)

interphase

occurs

copy

chromosome

throughout

The

RNA

a

of

composed

S of

the

period

phase, 2

of each

which

chromosome

identical

DNA

has

sister

chromatids

growth

after

replica

doubled linked

its at

DNA

the

NOTE centromere. Many

chemotherapeutic

targeting This

is

Some

agents

specific a

phases

frequently

tested

commonly

cell

cycle



they

S phase:

of

tested

function

the

area

cell on

agents

with

by



cycle. the

phase

preceding

exam.

phase

G2

(gap

2)

mitosis.

is

a

period

of

Replicated

cellular DNA

is

checked

for

DNA any

division.

of

target:

methotrexate,

5 fluorouracil,

M

hydroxyurea





G2

G0

phase:

M phase:

bleomycin

paclitaxel,

G2

G1

vincristine,

vinblastine S •

Non

cell

errors

cycle

cyclophosphamide,

specific: cisplatin

Figure

I 1 Figure

2.

The Eukaryotic I 1 2. Eukaryotic

Cell

Cell Cycle

Cycle

synthesis before

but cell

to

CHAPTER

Control by

of

the

strategic

points lar

cell

cycle

proteins

ensure events

that in

Reverse

the

is

cells

will

tion

also

ing

certain

life

through

a

occurs

to

a

highly

and

enter

the

cycle

of

acids

(an

extent

repetitive

(DNA

components:

a

in

in

RNA)

nitrogenous

Carbon

the

AND are

base,

5

of

various

until

RNA,

is

and

DNA

(Chapter

plays

ACID

STRUCTURE

more

commonly their

a

transcrip

role

in

amplify

7).

NOMENCLATURE from

carbon

nucleotides,

sugar

(pentose),

nucleotides)

are

which and

consist

of

3

phosphate.

Sugars HY

HY Nucleic the

acids

(as

pentose

well

they

nucleic

acid);

nucleic

acid).

as

nucleosides

contain.

if

the

If

pentose

and

the

pentose

is

deoxyribose,

is

ribose, the

classified

the

nucleic

nucleic

acid

according

acid is

DNA

are

(ribo MY

LY

2 types and

of

nitrogen

containing

bases

commonly

found MEDIUM in nucleotides: YIELD

Pyrimidines O

NH2 N

N

N

H2N

H

N

N

N

YIELD

Figure

I 1 3. Figure

contain

nucleic RNA.

include

2

acids

xanthine,

Pyrimidines Thymine

found

only

only (T)

in

their

and

metabolites,

RNA.

is

1

and ring.

usually

The

guanine usually

uric

Cytosine found

N

Thymine

Acids

commonly

found

both

are

found

in

found

in

nucleic

DNA

acids,

acid. (C)

only

purines (G);

REINFORCEMENT CH3

H

Found in Nucleic Found in Nucleic Acids

not

O

O

Uracil

is in

present

DNA,

in whereas

both

DNA uracil

YIELD

FUNDAMENTALS

H

structure.

(A)

hypoxanthine, have

RNA.

in

Commonly Commonly

adenine

purine

N

O

Cytosine

Bases

rings

are

Other

Bases I 1 3.

LOW

HN

H

Guanine

Purines

N

O

YIELD

REINFORCEMENT

NH

H

Adenine



YIELD

MEDIUM

FUNDAMENTALS O

NH2

HN

N

N

and

HIGH

pyrimidines.

Purines

in

LY

Yield

LOW



MY

(deoxyribo

High

purines

to

is RNA

Bases There

AND

molecu

Reverse it

NUCLEIC

check

the

express

provirus). where

|

phases

These

cycle

an

cells,

assembled a

the

replicate

integrated

the

kinases. of

copies

human

sequences

and

concluded.

which

STRUCTURE

Nucleic

phase

are

DNA

intermediate

between

dependent

next

phase

retroviruses,

limited

checkpoints

cyclin

produces

cycles

NUCLEOTIDE

Five

not cell

DNA

at

cyclins

which

with

genome

as

previous

transcription,

associated

accomplished

such

1

and (U)

is

ORGANIzATION

Immunology

PART

I

|

BIOCHEMISTRY

Biochemistry Nucleosides

and

Nucleosides sugar.

are The

Nucleotides

formed

by

numbers

covalently

identifying

in

nucleosides

and

or

pyrimidine

base.

linking

the

nucleotides

to

a

carbons

of

distinguish

base the

to

the

sugar

them

number are

from

1

labeled

the

carbon

of

with

carbons

of

a

“primes” the

purine

Medical Genetics

Nucleotides carbon

are of

when

a nucleoside.

compounds

Behavioral Science/SocialSciences

formed

1 or

more

Nucleoside

because

of

the

phosphate

di

hydrolytic

and

groups

is

triphosphates

energy

attached

are

associated

to

high

with

the

5′

energy

the

acid

anhydride

bonds.

Uridine

Monophosphate

Deoxyguanosine

Monophosphate

(UMP)

(dGMP)

O

O

ATP

N HN

HN

NH2 N High

energy

N

N

N O

O

N

O

O O

O–P P O

O–P

O

O

CH2

P

́ O

5

CH2

1 ́

3

OH

́

2

OH

Figure Figure a

I 1 6. High Energy I 1 5. High Energy Bonds Nucleoside Triphosphate side Triphosphate

Bonds in a Nucleo

P

́

CH2

O

O

O: ́

4

́

1 ́ ́

3

OH

́

2 ́

OH

in Figure

The shown etc., always

6

O

O

O

4

O–OH

N

O

O–5

O

N

H2N

bonds

nomenclature below.

for Note

is sometimes found

the

that

to

5. Examples I 1 4. Examples

commonly the

understood attached

I 1 Figure

“deoxy” and

deoxyribose.

found part not

of Nucleotides of Nucleotides

bases, of

expressly

the

nucleosides, names stated

and

deoxythymidine, because

nucleotides dTMP,

thymine

is almost

is

CHAPTER

Table

I 1

2.

Nomenclature

of

Important

Bases,

Nucleosides,

and

1

|

NUCLEIC

ACID

STRUCTURE

AND

ORGANIzATION

Nucleotides

Base

Nucleoside

Nucleotides

Adenine

Adenosine

AMP

(dAMP)

ADP

(dADP)

ATP

GMP

(dGMP)

GDP

(dGDP)

GTP

(dGTP)

CMP

(dCMP)

CDP

(dCDP)

CTP

(dCTP)

UMP

(dUMP)

UDP

(dUDP)

UTP

(dUTP)

(dATP)

(Deoxyadenosine)

Guanine

Guanosine (Deoxyguanosine)

Cytosine

Cytidine (Deoxycytidine)

Uracil

Uridine (Deoxyuridine)

Thymine

(Deoxythymidine)

Names

of

nucleosides

(dTMP)

and

nucleotides

(dTDP)

attached

to

(dTTP)

deoxyribose

are

shown

in

parentheses.

NOTE

NUCLEIC Nucleic

acids

that

is,

sugar

a

A

the

at

In

(left left



If



The



In

in

ssDNA

genomes

is

of to

the

3′

has

a distinct

found

at

often

a nucleic

right).

Figure

joined

1

6

by

carbon

of 5′

the

5′

of “d”

DNA

is generally

3′,

5′

a sugar end

and

end,

phosphodiester

to

the

3′

and

a

which

5′

5′

end,

and

hydroxyl



bonds;

carbon

of thus

the

has

group

the 3′

next

of

the the

5′→3′ strand

TCAG:

GACT

5′

pTpCpApG dTdCdAdG

in dsRNA

(dsDNA) certain

viruses,

genomes.

and

RNA some

is of

generally which

Acids

Nucleotides

linked

phosphodiester

by

3′,

5′

bonds

polar

is often

in

sequence

or

3′

shown:

stranded

have

convention,

labeled:

included:

occur

by

TCAG

be be

double

of

written

convention,

must

be

Exceptions some

this

may may

is

written

ends

phosphates

(ssRNA). and

be

the

(deoxy)

strand to

must

backward,

a

acid

According I

positions

stranded

nucleotides



end.

written

DNA,

of links strand

group 3′

eukaryotes,

single

Each

sequence

direction the

group

chain.

the

base

on

polymers

phosphate

found

The

are

phosphate

in

ity.

Nucleic

ACIDS

have



Have

distinct

thus

polarity

Sequence

3′

always

and

5′

specified

ends,

as

5′→3′

Immunology

PART

I

|

BIOCHEMISTRY

Biochemistry 5

́

Phosphate

3 ́

Hydroxyl

O H O

Medical Genetics

P

O

N

H3C

O

N

H

OH

O 5 ́CH2

T

5

N

N

H

A

N 3 ́

O

N

́

N

Behavioral Science/SocialSciences

3

O O

3 ́

5 ́CH2

O

O

O

P

H

O

O

P

O

N N

H

O

O

O C

H

N

5 ́CH2

N

G

N 3 ́

O

N

N H

O

N O

3 ́

H 5 ́CH2

O H

O

N N O

P

CH3

O

O

O

O

5

H

A

N

H

N

N

O

T

N

́CH2

P

O

N O

O 3

́

3 ́ O O 5

H O

P

N

O

H

O

́CH2

N

O

O O N

G

N

H

N

H

O

P

O

C

5 ́CH2 N

N

O

N

O

H

3 ́

3 ́

O 5 ́CH2 OH 3

O ́ O

P

O 3

́

Hydroxyl

5 ́

Figure Figure

I 1 I 1

6. Hydrogen 7. Hydrogen

Bonded Base Bonded

Pairs Base

in DNA Pairs

in

DNA

Phosphate

5 ́ O

́

CHAPTER

DNA

Structure

Some

of



|

NUCLEIC

features

of

2 strands

double

are

stranded

antiparallel

DNA

Using

include:

(opposite

in

In

direction).

The

2 strands

bonds),

are

and

sequence

G on

complementary.

always

one

A

pairs

strand

with

C

defines

always

(3

the

pairs

with

hydrogen

base

T

bonds).

sequence

(2

Because T,

of

and

total

ORGANIzATION

the

the

the

specific

amount

base of

pyrimidines.

G

pairing,

equals

These

the

the

amount

amount

of

properties

are

of C.

known

A

the

other

as

(or

minor

modification

(substitution

of

U

for

T)

these

base

the

total

stranded)

% T (%

U)

% A =

amount

purines

Chargaff’s

rules

dsRNA)

strand.

equals

Thus,

Rules

double

=

% C

of equals

% purines

=

%

pyrimidines

rules. A sample

With

AND

hydrogen

Thus,

on

=

% G •

STRUCTURE

Chargaff’s

dsDNA

(ds •

ACID

NOTE

the

The

1

also

apply

to

of

what

is

10%

G

DNA

the

has

10%

G;

% T?

dsRNA.

Most

DNA

Watson strand

are

in

Crick

each

occurs DNA

in turn

in

G

nature

DNA is on

stacked

complete

Z

occurs

the of

or

the

as

B

DNA.

outside

center the

of

the A

sequences

is unknown,

but

right

handed

The

of

helix.

C–rich

a

the

hydrophilic double

left

is known may

be

related

helical

sugar helix.

molecule. rare

double

There handed

The are

phosphate

about

Z

to

gene

bonded

10

base

helical

DNA.

The

known backbone

hydrogen

double

as

molecule

form

biologic

pairs of

of base

+

10%

C =

20%

as therefore,

% A +

% T must

total

80%

pairs 40%

A and

Ans:

40%

40%

T

per DNA

function

that

T

of

regulation.

AT AT CG GC

BRIDGE

TO

PHARMACOLOGY

AT TA

Daunorubicin

GC

drugs

and

that

are

doxorubicin

used

in

are

the

antitumor

treatment

of

TA leukemias.

Major Groove

TA GC

Provide for

CG

They

intercalating

regulatory

binding

sites

thereby

exert

their

between

the

interfering

with

effects

by

bases

the

of

activity

DNA, of

proteins topoisomerase

II and

preventing

proper

AT replication

of

the

DNA.

Minor Groove

GC

Other

drugs,

such

as

cisplatin,

which

is used

GC in the

AT

bind

TA

treatment tightly

distortion AT AT

Figure

I 1

8. The B DNA Double Figure I 1 7. B DNA Double

Helix Helix

of to

and

the

bladder DNA,

malfunction.

and causing

lung

tumors,

structural

Immunology

PART

I

|

BIOCHEMISTRY

Biochemistry Denaturation

Double

stranded

DNA

Double

helical

bonding

and

single Medical Genetics

Denaturation

and

(heat)

Heat,

Denatured

Single

stranded

separate

DNA

is

again

(Figure

I

1

in

two

covalent

such

be

as

hydrogen

double

bonds

helix are

formamide

renatured

example,

if

and

(annealed)

a solution

complementary

to

then

Chapter

a mixture

of

renatured.

of blot

7).

In

target

When

complementarity,

stranded

annealing

a Southern

in

added

into

broken urea

two in

this

are

if

the

denaturing

containing

strands

can

heat

become

denatured

base

paired

these

the

in

is

binds

a well The to

strands

the

molecules.

DNA

process

DNA

performing

techniques,

DNA

probe

complementary

and

target

called

hybridization.

portion

of

is

an

polymerase

important

chain

characterized

reaction

probe

mixed

sample

DNA

sequences

DNA

is denatured of

is and

sufficient

DNA Recall

Denaturation I I 1 18. 9.Denaturation Renaturation Renaturation

can

For

the

or

probing

(reviewed

(cooling)

and

No

the

8).

renaturation

step

Renaturation

Figure Figure

other.

disrupt

of

DNA Such

Double

that

“melting”

chemicals

DNA

removed.

cooled,

conditions

the

DNA.

stranded

slowly

by in each

and

denature

slowly

DNA

denatured

from pH,

to

single is

be

of

resulting

alkaline used

condition

can stacking,

that

commonly

Behavioral Science/SocialSciences

DNA base

strands

process.

Renaturation

of

of DNA

Question

and Methotrexate

DNA

affects

A.

G1

phase

B.

G2

phase

C.

M phase

D.

S phase

which

the

cell

cycle?

Answer:

ORGANIzATION

Large cell

OF

DNA and

molecules

still

be

D

DNA

must

be

packaged

in

such

a way

that

they

can

fit

inside

functional.

Supercoiling Mitochondrial

DNA

structures. in

These

which

results

the

helix

from

and

the

molecules is

strain

twisted

on

DNA may

the

of

as

around molecule

most

exist

itself

prokaryotes relaxed in

caused

are

circles 3

closed

or

as

dimensional

by

under

space. or

circular

supercoiled

overwinding

structures

Supercoiling the

double

helix:



Negatively than

supercoiled

in

Watson

in

Watson

DNA Crick

is

DNA.

formed

This

if

form

the is

DNA

is

required

wound

for

most

more

loosely

biologic

reactions. •

Positively than

1

supercoiled

DNA Crick

DNA.

is

formed

if

the

DNA

is

wound

more

tightly

the

CHAPTER



Topoisomerases in

are

DNA

enzymes

molecules.

alternately

breaking

example,

in

introduce

make

and

can

coli,

amount in

sugar

gyrase into

the

breaks

the

DNA

supercoiling

and

change

transient

resealing

Escherichia

negative

Nucleosomes

that

They

of

DNA

|

NUCLEIC

ACID

STRUCTURE

supercoiling HY by

strands

phosphate (DNA

1

backbone. II)

MY

can LY

DNA.

High

Chromatin

LY

HIGH

Yield

MEDIUM

ORGANIzATION

HY

For MY

topoisomerase

AND

YIELD

YIELD

MEDIUM

YIELD

Sensitive Without

+H1

30

H1

LOW YIELD nuclease

to

10 FUNDAMENTALS

nm

LOW

nm

FUNDAMENTALS

REINFORCEMENT

H2A

YIELD

REINFORCEMENT

H3 H2B

H2B

H3

H4

Expanded

H4

a

view

of

nucleosome

H1

Expanded

H2A

view

Figure

Figure

I 1 9. Nucleosome I 1 10. Nucleosome Structure Structure

Nuclear

DNA

nonhistone





in

Histones the

proteins.

Two

copies

DNA (a



is



30

nm

and

associated

chromatin

histones

associated

unit

and

of

arginine,

H2A,

DNA

which

H2B,

with

chromatin

H3,

is

confer

and

a

H4

histones

the

and

nucleosome.

positive

charge

aggregate

to

on

form

the

the

referred is

outside

is to

associated

as

a

with

them

into

of

this

sometimes

a

octamer

called

10nm the

to

“beads

chromatin linker

solenoid

a

a

nucleosome

string”

but

is

fiber).

DNA like

form on

found

structure,

between

nucleosomes

which

is

a

thick

fiber. condensation

occurs

chromosome stranded

interphase for

lysine

around

H1

Further

in

in

packaging

nucleosomes

package

double

available

of

wound of

Histone help

in

each

eukaryotic

Cells

rich

properly

to

basic

Nucleofilament and Nucleofilament

octamer.

series

more

is found

The

are

histone •

eukaryotes

proteins.

and

in Eukaryotic DNA in Eukaryotic

2

types

expression) with

G0

eventually or

G1

form contains

the one

chromosome. linear

Each

molecule

of

DNA.

contain

gene

to

in

areas

and of

the

of

chromatin:

euchromatin

heterochromatin chromosomes

(much that

are

(more more

not

highly

opened

and

condensed

expressed).

1

Immunology

PART

I

|

BIOCHEMISTRY

Biochemistry More

Less

active

active

Medical Genetics

Behavioral Science/SocialSciencesDNA

double

helix

10

nm

chromatin

30

(nucleosomes)

nm

chromatin

30

(nucleofilament)

nm

to

fiber

forms

scaffolding

loops

attached

Higher

proteins

Euchromatin

Figure

Heterochromatin

Figure I 1 10. DNA Packaging I 1 11. DNA Packaging

Euchromatin with

condensed,

an

corresponds

each

other

producing

of

mitotic

nucleolus

Chapter

is

nucleosomes

nm

fibers).

(10

The

figure

as

below

region

fibers)

as

an

electron

chromatin

for

and

ribosome

assembly

of

a nucleolus. (discussed

3).

Euchromatin

Heterochromatin

Nucleolus

Figure

During sister

mitosis,

all

chromatids.

structure

is

visible.

chromosomes banding translocations,

1

the This

by

I 1 12. I 1 An Figure 11.

DNA

is

is the

only

highly

Chromosome karyotype

techniques

condensed

time

in

the

or inversions,

allow

cycle may

(metaphase prometaphase), and

Nucleus Nucleus

to cell

abnormalities analysis

(prophase deletions,

Interphase An Interphase

be

assessed

chromosomes) which

duplications.

separation when

identify

highly

character

micrograph

heterochromatin,

specialized

loosely is more

well

shows

euchromatin,

nm

Heterochromatin

heterochromatin

containing

a nuclear

the

30

interphase

nucleus

Cell

to

(looped

chromosomes.

interphase

The

in Eukaryotic Cell in Eukaryotic

generally

associated

istic

order

packaging

the

of chromosome on

mitotic

and

by

aneuploidy,

the

in

CHAPTER

Review Select

1.

|

NUCLEIC

ACID

STRUCTURE

AND

ORGANIzATION

Questions the

A

ONE

best

double

with age

2.

1

answer.

stranded

RNA

gastroenteritis of

guanine

A.

15

B.

25

C.

35

D.

75

E.

85

What

is

the

genome

was in

found

this

structure

to

isolated

from

contain

15%

a virus uracil.

in

the

What

is

stool

of

the

percent

a

child

genome?

indicated

below?

NH2

N N

N

5 ́

N

CH2OH

4

O

1 ́ ́

3 ́

2 ́

OH

3.

A.

Purine

B.

Purine

C.

Pyrimidine

D.

Purine

E.

Deoxyadenosine

nucleotide

nucleoside nucleoside

Endonuclease features

activation of

mosome

OH

eukaryotic

structures

A.

Barr

B.

10

nm

fiber

C.

30

nm

fiber

D.

Centromere

E.

Heterochromatin

and cell would

chromatin

death

by

most

likely

fragmentation apoptosis. be

are

Which degraded

of first

characteristic

the in

following an

apoptotic

chro cell?

body

1

Immunology

PART

I

|

BIOCHEMISTRY

Biochemistry 4.

A

medical

her sample

What

to left

analysis Medical Genetics

student

mentor

behind

show is

working determine

the

by

10% most

a

likely

A.

Bacterial

chromosome

B.

Bacterial

plasmid

C.

Mitochondrial

D.

Nuclear

E.

Viral

source

genome

biology

composition research

40%

chromosome chromosome

a molecular base

former

adenine,

Behavioral Science/SocialSciences

1

in the

the

an

technologist.

cytosine, of

laboratory of

nucleic

30%

The thymine

acid

is

unlabeled

in

results and

this

asked

by

nucleic

sample?

20%

of

acid her

guanine.

CHAPTER

1

|

NUCLEIC

ACID

STRUCTURE

AND

ORGANIzATION

Answers

1.

Answer: U

=

C. A

Since

= A

15%. +

G

=

Alternatively,

2.

3.

+

G

=

G

=

35%.

D.

=

35%.

=

15%,

nucleoside

then

U

consists

adenosine,

B.

The

+

nm

of

which

more

10

endonuclease

A

=

30%

is

“opened”

the

fiber,

without

would

attack

a

base

the

a

sugar.

adenine

The

figure

attached

DNA,

the

H1,

is the

most

region

of

unprotected

the the

and

base

more

shows

to

sensitive

it

open

ribose.

is

to

structure DNA

enzyme listed.

between

nucleosomes.

Answer: rules

E. (%A

A =

stranded,

base %T,

nucleic

circular Only

A

The

The

A

G

and

nucleoside

Answer:

the

=

70%,

Answer:

attack.

4.

U

C

the

50%,

(choices a

few

viruses

compositional %C

=

acid A,

analysis

%G)

molecule. B

(e.g.

and

that

is indicative All C)

parvovirus)

or

of options

linear

single

from

Chargaff’s

stranded,

listed

(choice have

deviates single

except D)

DNA

stranded

not E

are

double

double examples

of

helices.

DNA.

1

DNA

LEARNING

Explain



Know

key



Know

major

Repair

2#

how

DNA

steps

and

in

RNA

DNA

kinds

of

synthesis

differ

replication

DNA

repair

REPLICATION

Genetic

information

parental

DNA,

are

identical

each

2

complementary

is

then

used

is transmitted a

of

the

process to

in the

a template replication).

2 identical

DNA

from

which

parent

2 daughter

parental

strands as

(semiconservative one

and

OBJECTIVES



DNA

Replication

DNA

of

parental

for

the

molecule. DNA

synthesis During

to

a

by

molecules During

are of

cell

progeny

DNA

DNA

pulled

apart.

new

complementary

division,

replication are

each

of

produced

replication, Each

daughter

parental

that the strand

strand cell

receives

molecules.

1

Immunology

PART

I

|

BIOCHEMISTRY

Biochemistry Replication The

of

process

of

Prokaryotic DNA

and

replication

Eukaryotic

in

Chromosomes

prokaryotes

and

eukaryotes

is

compared

below.

Medical Genetics

Prokaryotes Origin

Eukaryotes

of

replication

Multiple

origins

of

replication

Behavioral Science/SocialSciences

Centromere

Sister

chromatids

separated

Figure

I 2

1.

SemiFigure Conservative, I 2 1. DNA

DNA

Replication

by

Bidirectional Replication by a Semi

Bidirectional

are

during

mitosis

a

Mechanism Conservative,

Mechanism

NOTE •

Polymerases

are

synthesize

enzymes

nucleic

phosphodiester

that

acids (PDE)

by

The

forming

bonds.

bacterial a single

creates

2

around •

Nucleases PDE



are

enzymes

that

hydrolyze

bonds.

from

remove

the

5′

or

the

nucleotides 3′

end

of

nucleic

acid.

the –

Endonucleases nucleic fragments.

1

acid

cut and

within release

the nucleic

circle.

meet,

a closed,

double

replication.

stranded

Separation

that

Replication

eukaryotic

move is,

resulting

chromosome

origins

replication

of

away

thus, in

a

the

of from

circular

the

each

2

DNA

parental

other

bidirectional of

2

molecule

strands

in

of

opposite

process.

production

The divides

2

of occurs

identical during

contains

replication.

forks

production

replication acid

forks

eventually

multiple of

is of

The

identical

DNA

directions 2

replication

circular

molecules

DNA.

Each a

origin

replication the

forks of

Exonucleases

chromosome

having

2 in

sister mitosis.

produced

at

identical

linear

the

one

Bidirectional

nucleus

chromatids

each

origin. molecules

during are

linear

molecule occurs

Completion

of

of the

separated

of

replication

dsDNA

S phase from

by the

(sister of each

the

dsDNA

of

process

a

results

chromatids).

eukaryotic other

having means

when

pair in DNA

cell the

cycle. cell

CHAPTER

The shown

structure

of

a

representative

eukaryotic

chromosome

during

the

cell

cycle

2

|

DNA

REPLICATION

AND

REPAIR

is

below.

Panel

A

Cell division

M

G2

G1

S

Centromere

2 (sister

ds

DNA

ds

DNA

chromatids)

Panel

B

3 p

2 1

1

2 q 3 4

Drawing

Panel

B:

a

Drawing

of

a

chromosome

chromosome

chromatids

(metaphase)

difficult

Different

I 2 2. Panel

Panel A:

replicated

Photograph

stained

I Figure 2 2.

Figure

of

replicated

A: Eukaryotic Eukaryotic

DifferentRepresentations Representations

of

chromosome.

Chromosome Chromosome

a

stained

The and to

centromere

visualize

in

Replication Replication During

of a ofReplicated a Replicated Eukaryotic

replicated

individual

the

are photograph

S Phase During Panel

Chromosome Eukaryotic

B:S

Phase

Chromosome

1

Immunology

PART

I

|

BIOCHEMISTRY

Biochemistry COMPARISON

The

overall

RNA.

process

These

RNA

Medical Genetics

OF

2

of

types

polymerases,

DNA

DNA

of

AND

replication

nucleic

acids

are

́ C

A

T

5 ́

G

A

the

synthesis

synthesized

by

of

DNA

both

DNA

polymerases

and and

respectively.

RNA

Polymerase

Polymerase DNA

3

SYNTHESIS

requires

DNA Behavioral Science/SocialSciences

RNA

A

C

C

U

G

T

A

3

́

G

C

C

G

DNA

Template

A

A

C

T

C

Primer

RNA

T

G

G

A

required

DNA

synthesis

using

dNTP

3

5 ́

́ C

A

T

G

A

C

T

A

G

C

C

Template

G

A

for (5

A

C

T

Primer ́→3

́)

substrates

C

not

T

G

G

A

(5

́→3

required

RNA

synthesis

using

NTP

5

́

for ́)

substrates

primer

3 ́ C

A 5

́

T A

G C

A U

C G

T

A

A

T

G

C

C

G

C

G

A

G

T

3 ́

A

C

T

C

T

G

G

A

5

3 ́

́

C

A

T

G

A

C

5

Mispaired

́

T A

A

G

C

C

G

U

C

G

G

U

A 3

deoxynucleotide

removed

(3

́→5

A

C

T

C

Mispaired

́ exonuclease)

T

G

G

A

5

́

G

A

5

́

A

5

́

not

nucleotide

removed

dTMP

3

́ C

A

5 ́

3

́ C

A 5 ́

T

G

A

C

T

A

A

C

U

G

A

T

T

G

A

C

A

C

T

U

G

A

G

C

C

A T

G

C G

G 3

A

A

I 2

1.

G

C

C

G

A

C

G

G

C

T

Comparison

T

C

T

G

G

A

5 ́

3

acid

of

A T

DNA

C G

T A

C G

I 2 Figure

and

T A

G C

G 3

A

5

3 ́

3. I 2 Polymerase 3. Polymerase

RNA

Enzymes Enzymes

2

U

G C

C

C

G

polymerases

T

G

Synthesize DNA Synthesize

A

T

A

G

5 ́

C

A

U

C

G

and

DNA RNA

C G

G

A

U

3 ́

C G

A

C

3′→5′)

exonuclease)

require

Polymerase

dGTP,

(or

dCTP,

dTTP

DNA)

RNA

templates.

synthesis

A

A

C

T

C

T

G

U

G

A

G

A

C

Polymerase

CTP,

UTP

No

These

enzymes

are

most

commonly

G

fidelity

None

Yes

T

U

RNA

GTP,

C

G

RNA

ATP,

T

U

and

DNA*

RNA

(3′→5′

A

DNA*

primer

RNA

A

RNA

Required

and

A

DNA

dATP,

DNA

T

Polymerases

(5′→3′)

(copied

́ C

́

substrates

viruses.

C 5 ́

RNA

Required

with

A

Low

template

*Certain

G

synthesis

Required

activity

T

fidelity

synthesized

Proofreading

A

DNA

DNA

Nucleic

́ C

High

Figure

Table

C

́

associated

G 3

́

́

CHAPTER

Similarities

between



The

newly



The

template



The

newly

is

is

in

strand

is

added

when

DNA

REPLICATION

AND

REPAIR

include:

made

scanned

synthesized

nucleotide

reacts

template

is

with

a

strand.

during

this

Differences

in

the

the

5′→3′

3′→5′

direction.

direction.

complementary

and

antiparallel

to

the

nucleoside

the

3′

hydroxyl

triphosphate,

Pyrophosphate

(PPi,

group

which the

last

the

dNTPs,

is

two

of

base

the

growing

paired

with

phosphates)

is

the

released

reaction.

include:

The

substrates

for

RNA



DNA



DNA

for

whereas a

polymerases

whereas

the

substrates

RNA

contains

primer,

uracil.

whereas

cannot

RNA

initiate

polymerases

strand

do

synthesis,

not.

whereas

RNA

can. polymerases

can

correct

cannot.

mistakes

DNA

(“proofreading”),

polymerases

have

3′

whereas



5′

RNA

exonuclease

activity

proofreading.

STEPS

OF

DNA

molecular

1.

The

2.

Helicase

REPLICATION

mechanism

is as

of

DNA

replication

is shown

below.

The

sequence

of

follows:

base

the

are

NTPs.

require

DNA

polymerases

events

the

thymine,

polymerases DNA

synthesis

are

polymerases is,

for

DNA

synthesis

contains

That

The

synthesis

strand

strand

new

strand



RNA

|

strand.

Each



and

synthesized

template •

DNA

2

sequence

at

breaks

two

the

parental

the

origin

of

hydrogen

strands

replication

bonds of

DNA

is

holding to

recognized.

the

begin

base

pairs

unwinding

together.

and

forms

the

single

This 2

allows

replication

forks. 3.

Single

stranded

tion

of

them 4.

DNA

each

DNA

from

Primase

as

strand

a at

a

for

template are

from

the

III

at

the

3′

complementary strand ing •

be

each

to

stranded

reassociating

and

of

por

protecting

each

are

DNA,

in

The required

and

the

5′→3′

parental

strand

because

can

only

DNA

extend

a

“primer.”

synthesizing

of

primer

strand.

primers

synthesis

a preformed

RNA

parental

RNA

initiate

DNA

RNA

antiparallel

made

binds from

nucleotides)

on

process.

begins end

and

can

this

of

10

origin

to

end

polymerase

beginning

(about

the

unable 3′

(SSB) them

nucleases.

short

beginning

polymerases

DNA

by

synthesizes

is used

protein

preventing

degradation

direction,

5.

binding

strand,

primer.

to

continuously

the in

in The

parental

one

long

the

5′→3′

newly strand

piece

direction,

synthesized used

as

is

known

and

strand a

is

template. as

This

the

“lead

strand.” The

“lagging

strand”

fragments Each

(about Okazaki

primase, merase

and III.

is 1,000

synthesized

fragment then Each

is

long)

initiated

completed fragment

discontinuously

nucleotides

by is

made

by the

the

the

as

synthesis

synthesis in

as

known

of 5′→3′

a

series

Okazaki of

DNA

an using

of

small

fragments. RNA

primer DNA

by poly

direction.

2

Immunology

PART

I

|

BIOCHEMISTRY

Biochemistry •

6.

Medical Genetics

7.

Behavioral Science/SocialSciences

There

is

forks

on

the

RNA

primers

DNA

polymerase removes

ning

at

Both

eukaryotic their

makes

a of

DNA

9.

of

of

by

proteins

that

Eukaryotic •

DNA DNA

polymerase DNA,

the

I

begin

ability

to

If DNA

unpaired

synthesis

“proof

polymerase

base

at

the

3′

continues.

fragments,

converting

resealing

block

the

action

of

gyrase.

in

them

to

DNA

in

the

are acid

a

kills

topoisomerase

anticancer

II

agents.

to

be

worked

prokaryotes

DNA

Quinolones

eukaryotic

rep DNA

very

out.

are

similar

to

steps

and

with

those

The

compared

this.

used

2.

Polymerases α

lagging

and

δ

work

together

to

synthesize

both

the

leading

strands.



DNA

polymerase



DNA

polymerases

repair.

DNA

γ replicates β

mitochondrial

and

ε are

polymerase

thought

ε may

HY

DNA. to

participate

substitute

for

primarily

DNA

in

polymerase

δ

DNA MY in

certain LY

cases.

High

NOTE

Telomerase

Telomerase

Telomeres

are

eukaryotic

chromosomes.



Completes

the

telomere

replication

sequences

eukaryotic

of

at

both

the ends

telomeres of

chromosome

a

5′

the

telomeres

and •

Present and

in embryonic certain

present

adult in

cells, stem

adult

fetal

cells;

somatic

not

are

the

end

the

Inappropriately cancer unlimited

2

cells,

present contributing replication

in

many to

their

sequences

each

each

because

strand.

become

cells

at

With

shortened

of

Telomerase

DNA

This so

the

ends

round

that

linear

the

to

DNA

replication

polymerase

contributes

short

of of

cannot the

aging

chromosomes

molecules MEDIUM

in

most

cells,

cannot

H

in YIELD

normal

ME

cells,

the

LOW YIELD synthesis of

complete of

die.

telomere

an

stem

enzyme

template reverse

eukaryotes to activity

would

is present but

used

complementary

that

activity cells,

in

transcriptase

sequences

telomerase and

is RNA

telomerase •

repetitive

Yield

because

eventually

function properly FUNDAMENTALS

FUN

cells,

a short

cells

a

gyrase

passing

Nalidixic

believed

completely

replication

I 2

is

been

each the

DNA

DNA,

strands.

of as

of

forks,

topoisomerases.

useful

front

form. of

both

eukaryotes

yet

in

replication

strands

Inhibitors

becoming

“swivel”

the supercoils

both

then

not

a at

positive

nicking

and

polymerases

and

resulting

DNA

nick,

are

Table

uncharacterized

new

have

provides

and

replication

in in

an DNA

activity.

Okazaki

the

by

have

involved

eukaryotes

replication

fragment.

the

II)

DNA

details

and

synthesizes

before

unwinds

inhibiting

of the

two

prokaryotes

exonuclease

synthesis,

overwound

the

mechanism

In

polymerases

3′→5′

topoisomerase

teniposide)

However,

a

between

supercoils

(etoposide,

the

DNA.

becomes

drugs

bacteria

of

eukaryotes

Okazaki

is removed

helicase

through

family

each

and

DNA of

“nicks”

(DNA

in

DNA.

exonuclease)

DNA

of

negative

strands

for

H

with

prokaryotic

the

As

RNAase

neighboring

strand

seals

it

(5’

during

fork.

inserts

strand

gap

means

growing

gyrase

the

the

by

strand

ahead

in

of

mistake

lication

The

end

and

ligase

DNA

in

primer

work

continuous

lagging

by

fills

3′

the

a

removed

the

the

read”

and

chromosome. are

both

end 8.

a leading

not

in

otherwise only

somatic

in cells.

to

the

DNA

(hTRT). be embryonic

maintain

the

telomere

sequence,

Telomerase lost

during cells,

REINFORCEMENT telomeres. It contains

is thus replication.

germ

(reproductive)

as able

well to

as replace

Normally cells,

RE

CHAPTER

Cancer

cells

telomeres

often

have

from

relatively

becoming

high

levels

shortened

and

of

telomerase,

preventing

contributing

to

the

the

2

BRIDGE

immortality

|

TO

AND

REPAIR

PHARMACOLOGY and

fluoroquinolones

inhibit

DNA

cells. gyrase

Table

REPLICATION

of Quinolones

malignant

DNA

I 2

2.

Steps

and

Proteins

Involved

in

DNA

(prokaryotic

preventing

Replication

These Eukaryotic

drugs,

in

Origin

Replication

of

Prokaryotic

replication

Unwinding

of

(ori)

DNA

One

double

Cells

ori

site

replication

which

II),

and

are

most

transcription.

active

against

Cells aerobic

Step

topoisomerase

DNA

gram

negative

bacteria,

include:

(Nuclei)

per

Multiple

ori

chromosome

chromosome

Helicase

Helicase

sites

per



Levofloxacin



Ciprofloxacin



Moxifloxacin

helix Resistance Stabilization

of

template

unwound

Single

strands

stranded

DNA

binding

protein

Synthesis

Synthesis

of

RNA

primers

of

DNA

Lagging

strand

merase

protein

(SSB)

polymerase

III

binding

time;

DNA

DNA

polymerases

poly

δ

DNA

III

+

δ

α

polymerases

of

primers

DNA

polymerase

(5′→3′

of

RNA

with

and

infections

in

BRIDGE

TO

I

RNAase

exonucle

the

H

(5′→3′

exonuclease)

polymerase

I

DNA

polymerase

DNA

of

positive

supercoils

ahead

ligase

advancing

replication

DNA of

DNA

topoisomer

ase forks

of

ligase

DNA

azido

Once to

as

topoisomerase

II

in

genome.

The

replacement

normal

hydroxyl

telomeres

Not

required

the

Telomerase

by

High

transcriptase

HIV,

use

is to

this

in

cells



Associated



Encoded

tive

sequences

to

RNA

also

replicate

reverse

telomerase

human in

DNA

be

DNA new

polymerase

DNA.

RNA

genomes.

inhibited

by

transcriptase

most

DNA

AZT,

ddC,

is

related

enters

cells,

triphosphate for

it can

be

derivative the

viral

synthesizing

reverse

DNA

from

azide

instead

at

3′

the

and

its

of

position

a

MY prevents

deoxyribose effectively

causing it is

chain LY

(see

that Chapter

YIELDan

of

YIELD reverse

FUNDAMENTALS

play

a role

genomes in

amplifying

polymerase, proofreading

replication

termination. reverse activity.

YIELD

MEDIUM

YIELD

LOW

YIELD

ddI.

FUNDAMENTALS

REINFORCEMENT

viral

a DNA lacks

HIGH

further

notably LOW by

synthesis and

activity:

(residual DNA)

Yield

that requires MEDIUM

Retroviruses,

(hTRT)

retrotransposons in

of their

can

contain

with

dependent

synthesis

retroviruses

by

maintained

an the

enzyme

transcriptase

Eukaryotic

direct

HIV

gyrase)

Transcriptase

template

of

structurally

of an HY group

HY

II (DNA

transcriptase

RNA

tract

2′,3′

AZT

the

a substrate

RNA

Although

Reverse

urinary

treatment (3′

LY

Reverse

lower

sexes.

or

transcriptase

MY Synthesis

over of

PHARMACOLOGY

of AZT

converted

δ

fragments

Removal

both

compounds.

DNA

Okazaki

developed treatment

and

chemotherapeutic use

used of

upper

dideoxythymidine)

DNA

Joining

has

include

+

ase)

Replacement

gonorrhea

One

RNA

drugs

uses

α

fragments)

Removal

the

current

Primase

DNA

strand

to

stranded

DNA

(SSB)

Primase

Leading

(Okazaki

Single

REINFORCEMENT

permanently certain

repeti

7)

2

Immunology

PART

I

|

BIOCHEMISTRY

Biochemistry 5

́

3 ́

3

́

5 ́

Leading 1.

Strand

Primase

Synthesis

(Continuous)

synthesizes

the

primer

(

) 5

́

to

3

́.

Helicase 2.

Medical Genetics

DNA

polymerases

into

Origin 3.

the

Helicase

Lagging

Behavioral Science/SocialSciences

and

δ

fork

(

) continues

Strand

Synthesis

extend

the

(Leading to

primer,

strand unwind

moving

synthesis).

the

DNA.

(Discontinuous)

3 ́

5

3

α

replication

́

5 ́ ́

1.

Primase

2.

DNA

synthesizes polymerases

away 3.

α

from

the

Synthesis

stops

the

of

primer

of

the

the

primer

and

δ

DNA

leading

diagram,

or

the

fork

when

of

3

́. moving

strand

polymerase

primer

to

primer,

(Lagging

strand

the

) 5 ́

extend

replication

the

(

synthesis).

encounters

on

the

other

the

previous

side (Okazaki)

fragment. 4.

As

helicase

third

RNAase the

next

more

of

fragment

H RNA

(5

the

will

replication

be

from

cell, fragment

activity)

fragment

DNA

1.

In

polymerase

(2),

to

fork,

fill

in

digests the

extends the

a

added.

́ exoribonuclease

primer

eukaryotic

3 ́

5 ́

opens

Okazaki

the

gap.

5 ́

3 ́

In

prokaryotic

the

5

́

the

DNA

cells

(2)

both

to

types

fragments

of 1

polymerase

activity

polymerase

fragment In

DNA

exonuclease

to

activity fill

in

cells

and

2

the

to

has

both

primers,

extend

the

and next

gap.

DNA by

1 remove

ligase

making

connects a

phosphodiester

bond. 5 ́

3

́

5

́

5 ́

3

́

3 ́

5

́

3 ́ +

This

whole

primers

Figure Figure

I 2 4. I 2

4.

Question

Which

of

repeats

from

both

DNA

Replication

DNA

Recall

replication

process

the

following of

DNA

A.

DNA

gyrase

B.

DNA

helicase

C.

DNA

ligase

D.

DNA

polymerase

the

to

leading

remove and

all lagging

RNA strands.

Replication

enzymes

is

the

target

of

fluoroquinolones

in

strands?

Answer:

2

A

CHAPTER

DNA

BRIDGE

REPAIR

The

structure

of

chemicals tion.

or

of

their

DNA.

occurs

in

any

G2

bases

If

phase

to

also

of

correct

be

to

of

cells

exposure during

their

an

the

DNA

risk

cell

Mismatch

of

DNA

repair

may

tumor

not

I 2

3.

DNA

occur

properly

suppressor

inactivated

AND

REPAIR

PATHOLOGY

through



the

cancer.

Most

The

p53

when

genes

mutation

gene

prevents

repair

have

or

errors.

or

encodes

a cell

from

entering

been

deletion:

a

with the

deletion

protein

that

damaged

DNA

S phase.

associated

Fraumeni Table

REPLICATION

a

into

increased

DNA

|

TO

certain

sequence

using

mutations

cycle.

to

replica

maintain

stable

eukaryotic

replication

through

to

replicate

carries

the

ways

incorporated

introducing

repair

phase

of

allowing

allowed

risk

DNA

G1

number

can

are

high

in

the

a

evolved,

cells

is a

defect in

in

have

there

occurs the

damaged

Incorrect

genomes.

Thus

repair

be

systems

template,

DNA

can

repair

damaged new

DNA

radiation.

Multiple

stability

2

Inactivation

with

syndrome

and

Li

many

solid

Repair tumors. Recognition/

Damage

Cause

Excision

Repair Enzyme



Enzymes

ATM for

Thymine

UV

radiation

Excision

endonuclease

DNA

polymerase

DNA

ligase

DNA

polymerase

gene p53

ataxia dimers

(deficient

(G1)

in

xeroderma

encodes activity.

ATM

replication

A

mutation

on

one

of

(breast, matched

errors

two

base

genes,

or

(G2

and

hMSH2

hMLH1,

defective

S)

DNA

ligase

initiates repair

BRCA

2

The

retinoblastoma

first

tumor

and

is a

in

x rays

by

and

lymphomas.

BRCA

and

cancer)

(breast

ovarian

1

cancer).

Rb

gene

was

the

mismatches,

resulting tion

in

a

known

condi

as

hereditary sis

to prostate,

and

inactivated

of •

DNA

is

essential

characterized to

predisposition DNA

kinase

telangiectasia,

hypersensitivity

pigmentosum)

Mis

a

nonpolypo

colorectal

suppressor

gene

negative

cell

cycle

the

transcription

regulator

through

its

ability

factor

cloned, of

E2F

to

the bind

and

cancer—HNPCC. repress Cytosine

Spontaneous/

Uracil

glycosylase

deamination

heat

endonuclease

AP

DNA

polymerase

DNA

ligase

transcription

required

for

HY

G1

MY

LY

of

Thymine

Ultraviolet DNA

light (also

induces

the

occasionally

thymine

dimers

Thymine

formation

between

interferes

dimers

High

Dimers

are

of

other

with

pyrimidines).

replication

from

DNA

and by

The

normal

formation LOW expression.

gene

a nucleotide

excision

LY

HIGH

Yield

adjacentMEDIUM thymines YIELD

between

adjacent

DNA

eliminated

dimers

genes

S phase. HY

MY

Repair

of

in

YIELD

MEDIUM

YIELD

of YIELD

LOW

YIELD

repair

mechanism. FUNDAMENTALS Steps

in



nucleotide

An

excision

ter

backbone

and •

DNA

DNA

repair:

of the

the

damaged

fills

using seals

(excinuclease)

the the

makes

strand

defective

polymerase

ligase

REINFORCEMENT

endonuclease

removes

direction, •

excision

FUNDAMENTALS

on

both

nicks sides

in of

the

the

REINFORCEMENT

phosphodies

thymine

dimer

oligonucleotide. in

the

gap

undamaged nick

by

synthesizing

strand in

the

repaired

as

DNA

in

the

5′→3′

a template. strand.

2

Immunology

PART

I

|

BIOCHEMISTRY

Biochemistry Base

excision

Cytosine

repair:

cytosine

deamination

uracil.

The

uracil

deamination

(loss is

of

an

recognized

amino

and

group

from

removed

(base

cytosine)

converts

excision)

by

a

cytosine

uracil

to

glycosylase

enzyme. Medical Genetics •

Behavioral Science/SocialSciences

A

Subsequently

this

the

sequence

damaged



DNA

polymerase



DNA

ligase

summary

they

of

function

area

in

the

the

cycle

the

repaired

involved is

an

AP

endonuclease

that

removes

DNA

gap

in

genes cell

by

the

nick

important the

recognized

from fills

seals

in

is

in

shown

strand

maintaining

DNA

fidelity

and

where

below.

M

Mismatch repair • MSH2 • MLH1

G2

G1

G0 Thymine

dimer

(bulky S

lesion)

repair

• XP • Nucleotide

excision

repair DNA

polymerase

proofreads

(cytosine

deamination)

during

DNA

synthesis

Genes

controlling

entry

into

S

phase

• Rb • p53

Figure

Diseases

I 2 6. 5. Important Maintaining

Associated

Inherited with

Figure I 2

with

mutations

that

a predisposition

to

MY

Important Genes Associated with Genes Associated with Maintaining Fidelity Fidelity of Replicating DNA of Replicating DNA

DNA

result the

HY

High

Repair

in

LY

defective

DNA

development

of

mechanisms MEDIUM

repair

are

YIELD associated

extreme most

repair

replication.

2

sensitivity common

Hereditary to

pigmentosum

is an to

sunlight,

deficiency

mismatched

skin

occurs

nonpolyposis

autosomal

in

colorectal base

pairs

recessive

freckling the

and

DNA

disorder, ulcerations,

excinuclease

cancer in

ME

cancer. LOW

Xeroderma

H

Yield

results that

are

and

accidentally

a

skin

by cancer.

FUNDAMENTALS

enzyme.

from

YIELD

characterized

deficiency

FUN

REINFORCEMENT in the ability

introduced

The

during

RE

CHAPTER

XERODERMA

Xeroderma

pigmentosum

1/250,000) ulcerations, and

and

most

6

year

parents

and

creams

recessive

disorder

sensitivity

Carcinomas

to

and

The

most

Xeroderma

the

often

as

skin

melanomas

excision

should

Hereditary

appear

Hereditary

exposure

the

genes

early

in

occurs

These

deleted

of

the

occur,

causing

retain

errors

gene.

loss

of

life,

in

the

cell

of

the

in is

a

many

Patients

and

loci,

the

child’s

the

relevant

with

results

from

the

disease

a mutation

enzymes

remove

that

errors

or

one

in This

some

of

intestinal

the

causes

which

cells

out into

may

inherit

nonfunctional,

mutation

in

carry

introduced

individuals

function.

in

to

of

blood.

HNPCC,

somatic

other

manifested

area.

syndrome)

gene

repair

neck ointments

exposed

measurement

encoding

hMLH1

and

typical

was

(HNPCC)

with

facial

light.

detect

birth,

mismatch

This

UV

were

throughout

of

hMSH2)

families

After

the

(mutations)

progression.

undergoing

NOTE one

Microsatellites

prominent

instability.

other

copy

may

chromosomes

may

to

contribute

because

they

are

repeats)

to constant

a

are

repeats

of

patient

instability,

error with

whereas

used

repair) a family as

do

that

accompanies

HNPCC, normal

not

pedigree

a diagnostic

, tri

dispersed

usually

known

(but

, and

as

short

tandem

tetranucleotide

throughout

not

exclusively)

the in

DNA,

noncoding

regions.

DNA

cells cells

from

from

the the

replication resected

individual

is tumor (which

microsatellite show still

example,

particular micro

repair,

show and

tool.

microsatellite histologic

instability. analysis,

microsatellite

Along

with instability

TGTGTGTG locus.

the

may

If cells

replicated

occur

lack

at

a

in

the

mismatch

DNA

will

that

locus,

vary

retain number

mismatch

di

division.

type In

(also

deleted

For

from

REPAIR

tone.

(Lynch

enzymes

copy

hMSH2

of

parents

with

child

by cells

cancer

In

the

muscle

cancer

or

the

away

freckling

poor

white

colorectal

replication.

nonfunctional,

when

source

hMLH1

repair.

during

any

his

in go

diagnosed

colorectal

(usually

mismatch DNA

be in

to

nonpolyposis

of

and

because

not

excessive

stature

can

nonpolyposis

satellite

AND

freckling,

deficiency

blistering

did

worse

endonuclease

avoid

clinic

and

noted

slight

pigmentosum

enzyme

One

REPLICATION

(incidence

sunlight,

common

the

lesions

became

physician

well

to

lesions

that

and

as

brought

excessive

The

body,

cancer

autosomal

cancer.

was

noted

sunlight.

ly

child

with

The

copy

of

DNA

PIGMENTOSUM

extreme

cancer. die

old

concerned

the

skin

|

endonuclease.

A

DNA

an

by

patients

excision

one

is

characterized

2

of

repeats

at

e.g.,

information may

be

TGTGTGTGTGTG microsatellite

or

TGTGTG.

This

variation

is

instability.

2

Immunology

PART

I

|

BIOCHEMISTRY

Biochemistry Review Select

Medical Genetics

1.

Questions the

It

is

ONE

best

now

believed

stitutions

answer.

that

causing

bases

during

DNA

determining

substantial

proportion

genetic

mutation

are

Which

accuracy rate

of

disease

replication.

the

substitution

a

human

of

nuclear

in

human

the

due

to

single

proofreading DNA

nucleotide

sub

misincorporation

of

activity

is

and

thus

replication

critical the

in base

chromosomes?

Behavioral Science/SocialSciences

2.

A.

3′

to

5′

polymerase

B.

3′

to

5′

exonuclease

C.

Primase

D.

5′

to

3′

polymerase

E.

3′

to

5′

exonuclease

The

V.

a The

that

3.

inactivates

A.

TATA

B.

Cap

C.

Catabolite

D.

Acyl

E.

Single

The

is

box

deficiency to

lowing

of

impaired

expression

of

the

viral

protein

excision

endonuclease in

be

may

xeroderma

enzyme?

absent

in

produce

an

pigmentosum. a

patient

of

Protection

D.

Repair

of

mismatched

bases

during

DNA

E.

Repair

of

mismatched

bases

during

transcription

exquisite

Which

deficient

Removal

in

this

sensi of

the

fol

endonuclease?

introns pyrimidine

dimers

against

DNA

Pseudomonas

viruses

action

duplication

in

participates

in

bacterial

antibiotic?

A.

DNA

polymerase

I

B.

DNA

polymerase

II

C.

Topoisomerase

I

D.

Topoisomerase

II

E.

DNA

ligase

replication.

(SBP)

C.

this

DNA

enzyme

(CAP)

B.

by

HIV

encoded

(TBP)

of

enzymes

infection

designated

viral for

Removal

anti

a

required for

upon

virus,

(ACP)

would

chromosome

2

markedly

A.

The

δ

(CBP)

radiation

functions

III

polymerase

protein

protein

an

is

the

protein

binding

ultraviolet

polymerase

DNA

substrate

protein

strand

to

a potential

activator carrier

α

cells

nuclear

protein

γ

immunodeficiency

traced

binding

binding

T

human

δ

polymerase

DNA

of

cytotoxic

cell

DNA

of

activity

been

polymerase

polymerase

activity

a host

protein

tivity

4.

has

DNA

of

DNA

discovered

defect

Which

of

of

newly

of

activity

activity

proliferation

with

activity

of

norfloxacin

rapidly

is

dividing DNA

replication

replication

related

cells.

to

Which and

its of

is

ability the

directly

to

inhibit

following inhibited

CHAPTER

5.

Cytosine for

6.

arabinoside

cancer,

tain

cases,

nase

in

A.

cytosine

B.

cytidylic

C.

thymidine

D.

uracil

E.

cytidine

related

to

an

This

enzyme

congenital

(DKC)

capacity to

in

is

the

the

nuclei site

for

most

likely

centromeres

B.

Synthesis

of

Okazaki

C.

Synthesis

of

RNA

D.

Synthesis

of

telomeres

E.

Removal

Single

strand

DNA

damage.

damaging

of

These the

the

agent

develop.

enzyme

In

cytidine

araC

REPAIR

to

cer deami

form

is

ultimately of

enzyme

rapidly this

an

enzyme

in

cells.

contains

for

which

defect

Which

has

chromo

Structural a single

function.

DKC

in The

needed

dividing

protein

catalytic

disease

impaired.

analysis stranded

step

in

DNA

patients?

primers

in

DNA

comprise

breaks

are

by the

a

the

series

due

of

the

of

enzymes

phosphodiester

catalyzes

single

frequently

residues

repaired reform

class

of of

inherited

markedly

fragments

deoxyribose

break

DNA

in

AND

primers

RNA

breaks

is of

deficient

of

in

eventually

inactivate

genetically

cells

normal

Synthesis

Which

increase

production

A.

and

chemotherapeutic

may

would

is a stem

active

required is

of

inadequate

that

replication

of

effective

REPLICATION

arabinoside

that

type

an drug

DNA

arabinoside

shown

RNA

this

cells.

duplication

has

as

to

|

acid

traced

some

7.

is

tumor

proliferative

been

is used

resistance

resistance the

Dyskeratosis the

(araC)

although

2

the

most to

sugar

the

of

species

backbone.

reconstruct

between of

type

oxygen

phosphate that

bonds formation

frequent

reactive

This the

sugar

nucleotides. phosphodiester

bond

repair?

A.

DNA

glycosylases

B.

DNA

helicases

C.

DNA

ligases

D.

DNA

phosphodiesterases

E.

DNA

polymerases

2

Immunology

PART

I

|

BIOCHEMISTRY

Biochemistry Answers

1.

Medical Genetics

Answer:

2.

DNA.

otic)

do

with

DNA the

not

5′

to

5′

exonuclease

of

an

pol

participate

to

3′

E.

TBP

in

this

process,

and

CBP

participate CAP

ACP

is

Nucleotide

in

XP

4.

Answer:

D.

Norfloxacin

inhibits

5.

Answer:

D.

Deamination

of

6.

Answer:

D.

The

merase

required

for enzyme

This

the

ends

(telomeres)

C.

other

All

bonds of

DNA.

DNA primers are

eukaryotic

in

repair

the

of

human

pol

III

(prokary

are

replaced

always

gene the

fatty

of

is

DNA

synthesized

transcription

expression

acid

and

of

prokary

synthesis.

thymine

is of

(pyrimidine)

dimers

DNA

described

during

as

produce

an

RNA

duplication

linear

is

a

uracil.

dependent in

reverse

II).

the

DNA

nuclei

of

transcriptase,

poly

rapidly that

divid

replicates

chromosomes.

have

repair of

(topoisomerase

would

telomerase,

options

gyrase

cytosine

chromosome

backbone

phodiester strands

excision

enzyme

cells.

phosphate

represents

patients.

ing

Answer:

strands

regulates

involved

B.

7.

RNA

DNA

δ

replication

and

short

in

respectively.

operons.

the

new

pol

the

direction.

deficient

of

DNA

for

(mitochondrial)

and

Answer:

None

of

required

γ

replication,

translation, lactose

activity

enzyme

DNA

during

Answer:

otic

3

3′

chromosomal

mRNA

3.

The

activity

in

Behavioral Science/SocialSciences

E.

proofreading

reverse

systems

DNA. DNA

Although synthesis,

transcriptase

use

a

ligase

activity.

to

seal

polymerase these

breaks

in

enzymes enzymes

the

make do

not

ligate

sugar phos

Transcription

LEARNING

and

RNA

3#

Processing

OBJECTIVES



Use



Understand

concepts

of

prokaryotic



Understand

concepts

of

eukaryotic



Demonstrate

knowledge

of

types

of

RNA

understanding

primary

pre

mRNA

messenger

RNA

messenger

of

RNA

alternative

splicing

of

eukaryotic

transcripts



Know

key

features

of

ribosomal



Know

key

features

of

transfer

RNA

(rRNA)

RNA

(tRNA)

TRANSCRIPTION The

first

stage

in

information base

in

sequence

one

of

strand

of

merase moves is

as in

of

the

the

3′

to

single

along

below

the

illustrates on

stop

RNA.

of

For

strand)

strand

particular RNA

the only

poly

polymerase the

RNA

polymerase for

form gene,

by

DNA,

the

to

RNA

of

RNA

(terminators) an

any

Because

template.

of

molecule

is copied

direction.

template

the

transcription

DNA

product

recognizes

each

of

the

thousands

organism.

arrangement

and

direction

of

transcription

for

molecule.

Terminator

Promoter

3′

signals

genome

the

a DNA

of

to

the to

and in

5′

is

stranded

template

the

complementary

units

genes

(the in

direction

information

a double molecule

molecule RNA

5′

genetic of

stranded

(promoters)

figure

of

sequence

DNA

transcription

several

a

and

signals

The

expression base

it synthesizes

antiparallel

start

the the

Terminato

Promoter

Transcription

5

́ Spacer

Gene

DNA 3

Spacer

1

Gene

DNA

Spacer

2

Gene

DNA

3

́

5

́

DNA

́

Transcription

3 Spacer

Transcription

Promoter Terminator Figure

I 3

1.

Figure

Transcription I 3

1.

Transcription

of

Several of Several

Genes Genes

on on

a

Chromosome

a Chromosome

31

Immunology

PART

I

|

BIOCHEMISTRY

Biochemistry TYPES

OF

RNA

RNA

molecules



play

Ribosomal

Medical Genetics

cell.

a

variety

RNA

It

is

used

associates

of

roles

(rRNA), as

in

which

is

a structural

with

the

cell.

the

The

most of

proteins

to

types

abundant

component

ribosomal

major

the

form

of

type

of

ribosome.

the

RNA

are:

RNA

in

the

Ribosomal

complete,

RNA

functional

ribosome. •

Transfer

Behavioral Science/SocialSciences

Its

RNA

function

linked •

is

type

of

(mRNA),

very

heterogeneous

that

by

that

the



of



nuclear

of

RNA

information

ribosome.

of

they

will

RNA. be

mRNA

the

in

cell

thousands

has

of

the

RNA

population as

the

specifying Messenger

sequence, of

or

cells.

is

a

cell

the is

essentially

different

a

proteins

It

its

pre

mRNA),

represents

which

is

precursors

of

found

only

mRNA,

processing.

(snRNA), of

which

major

is

also

functions

only

in

the

participate

in

splicing

with

enzymatic

activity.

that

synthesizes

is

to

found

nucleus

of

mRNA.

which both

(hnRNA

posttranscriptional

introns)

Ribozymes, in

base each

RNA

RNA One

found

the

the The

and

eukaryotic

its

eukaryotes. (removal

to

for

nuclear

during

Small

type

where

cell.

nucleus

formed

abundant

ribosome,

carries

translated.

size

most

the

synthesis.

protein

molecule

Heterogeneous in

is

in

mRNA

a

second to

which

of

RNA

the

protein

sequence

only

made

are

RNA

molecules

prokaryotes

and

They

are

eukaryotes.

POLYMERASES

There

is a cell.

single

The

prokaryotic

core

subunit

structure of

transcription

of

transcription.

protein

are

RNA

they

3

RNA

factor.

5.8S

called

promoter.

binds

the

is

transcription

of

RNA

has

required

released

for

the after

requires is

in

the

immediately

polymerase

preventing

polymerases,

is

types

molecule

sometimes

RNA

DNA,

all RNA

(σ)

factor

prokaryotic to

the

sigma

Sigma of

The

RNA

I is

polymerase and polymerase

some

snRNA,

Transcription

factors

transcription.

The

mushrooms).

factor

making

a

inhibited

by

transcription.

distinguished

by

the

particular

types

of

II some

in

the

nucleolus

and

synthesizes

28S,

18S,

is

located

and (such

is

5S

located

the

nucleoplasm

and

synthesizes

hnRNA/

All

in

the

nucleoplasm

and

synthesizes

tRNA,

rRNA. as

TFIID

requirements

polymerase

in

snRNA. III

understood. RNA

located

rRNAs.

RNA

addition,

D

polymerase

RNA

well

a

Termination

(ρ)

eukaryotic

mRNA •

protein at

for

produce.

and •

A

Actinomycin

There



rho

polymerase

responsible

α2ββ′.

called

rifampin.

RNA

polymerase

initiation initiation

3

acids

during

acid

different

not

is

RNA

amino

the

which amino

carry

together

Messenger



(tRNA), to

for

for

transcription II

is

RNA

polymerase

termination can

inhibited

of be by

inhibited α

amanitin

II)

help

transcription by

to in

actinomycin (a

toxin

initiate eukaryotes D.

from

certain

are In

CHAPTER

Table

I 3

1.

Comparison

of

RNA

RNA

polymerase

ββ′)

Requires

sigma

initiate

at

(ρ)

to

(σ)

RNAP

1:

rRNA

RNAP

2:

hnRNA/mRNA

RNAP

3:

tRNA,

No

to

sigma,

promoter

bind

requires

No

a

Sometimes

AND

RNA

PROCESSING

(nucleolus)

rho

and

5S

but

before

Except some

rRNA

snRNA

rRNA

transcription

RNA

5S

factors

(TFIID)

polymerase

required

terminate

Inhibited

by

rifampin

RNAP

D

Actinomycin

Actinomycin

TRANSCRIPTION:

AND

TRANSCRIPTION

Eukaryotic

Single

rho

|

Polymerases

Prokaryotic

(α2

3

2

inhibited

by

α

amanitin

(mushrooms)

D

IMPORTANT

CONCEPTS

TERMINOLOGY

RNA

is

synthesized

template

for

transcription



RNA

RNA

which

its

The

polymerase

(uses

terminology

genes

begins,

which

moves

CTP,

DNA

used

DNA for

as

searching

as

when

the

of

template in

a

discussing

is

5′

DNA

is No

RNA

to

used

regions. establishes

as

primer in

3′

the

is

the

3′

direction

to

does

and

template,

required. 5′

direction

using

polymerase

complementary

promoter Binding

strand

the

substrates.

for

polymerase.

strand

product

product

by

RNA

proceeds.

along

RNA

UTP)

RNA

in site

transcription

the

The

RNA Important

binding

direction

GTP, work.

template

NTPs

not

antiparallel

proofread to

the

strand. coding

(antitemplate)

identical uracil By

the

synthesizes

(ATP,



locates is

polymerase

it

RNA).

below.

transcription in

dependent

of

polymerase

and



DNA

illustrated

promoter

where

as

a

synthesis

is

The



by

the

in

strand

sequence

instead

of

convention,

to the

the

the

thymine base

is

RNA

not

used

molecule,

found

sequence

in of

a

during

transcription.

except

that

It

RNA

is

contains

DNA. gene

is

given

from

the

coding

strand

(5′→3′). •

In

the

vicinity

location as –

the To

of +1 the

bases – •

To

Transcription

of

base left are

the

a gene,

important

right

of

that

(5′, –1,

a numbering bases.

gene

or –2,

(3′, ends

The

or

base

this

starting

is

used

to

transcribed

identify as

RNA

the is

defined

region.

upstream) –3,

system first

of

point

for

transcription,

etc. downstream)

when

RNA

of polymerase

this

point, reaches

bases

are

a termination

+2,

+3,

etc. signal.

3

Immunology

PART

I

|

BIOCHEMISTRY

Biochemistry Upstream

Downstream

Transcription

unit

Start Medical Genetics

Terminator

site

-10

+1+10

Coding

(antitemplate)

strand RNA

3

5 ́

́

Promoter 5

3 ́ Behavioral Science/SocialSciences

Template

DNA

RNA

polymerase

transcribes ́

DNA

template

strand

strand

RNA

5 ́

3

transcript

is

́ synthesized

5

́

3

́

HY

Figure

I 3 Figure 2. Transcription I 3 2. Transcription

of

MY

DNA of DNA

LY

Flow

of

Genetic

DNA

to

Protein

For

the

case

Information

High

of

a gene

found

in

double

stranded

below.

Messenger

RNA

and

antiparallel

mRNA carboxyl

in

from

the

5′

to

the

to

3′

coding

for

a protein,

DNA, is

the

single

synthesized

template

in strand

direction,

as

it

relationship

stranded

of

the

among MEDIUMthe

mRNA, 5′

DNA.

synthesizes

to

3′

and

The

ribosome

the

protein

sequences YIELD

protein

direction.

It

is

ME

illustrated

LOW YIELD complementary

is

translates from

H

Yield

the

the amino FUNDAMENTALS

to

the FUN

terminus. REINFORCEMENT

Coding

strand DNA

5 ́

DNA

A

T

GGGGC

T

CAGCGAC

coding to

(except

T

DNA 3 ́

TACCCCGAGTCGCTG Template

mRNA

5 ́

A

strand

is

3 ́ identical

Transcription

RE

5 ́

is

strand

the for

3

U)

template

strand

complementary

antiparallel

UGGGGCUCAGCGAC

mRNA

and to

Direction ́

the

mRNA

of

transcription Translation

Codons

Protein

NH2

Met

Gly

Leu

Ser

Asp

Direction

COOH

translation

Figure

3

I 3

3.

Figure Flow

I 3 3. Flow of Genetic

of Genetic Information Information

from from

DNA to DNA

Protein to Protein

of

CHAPTER

Review

|

TRANSCRIPTION

AND

RNA

PROCESSING

Questions

Select

the

1.

The

ONE

best

During

RNA

scribed

to

A.

ATCG

B.

GCTA

C.

CGTA

D.

AUCG

E.

GCUA

answer

Also

answer.

synthesis,

is

E.

DNA

which

RNA

is

that, of

of

template

the

and

convention, the

sequence

following

antiparallel

by

regardless

the

the

produce

remember

direction in

3

direction

TAGC

complementary

all

base

in

which

would

be

tran

sequences?

to

sequences the

are

the

template

written

sequence

in

may

strand. the

5′

actually

to

be

3′

used

cell.

Approach:



Cross



Look



What

out at

is

Examine

the

example)

at

usually

option 5′

end

the

complement

options end.

of

direction mRNA

(T

the

(RNA

has

in

this

case).

base?

(A)

this

correct

two

the

T

of

A

or

a

DNA

Repeat

one

Transcription

the

of

given.

3′

only

the

with

the

the

leave

2.

any

option

U).

will

procedure

have

for

the

the

3′

complement

end

of

the

(A DNA.

in

This

this will

options.

following

indicated

sequence

by

the

of

arrow.

the

What

tryptophan

would

be

operon the

base

occurs

sequence

in of

produced? 3′

CGTCAGC

5′

Transcription



Which

product?

5′...GCAGTCG...3′

The

5′...GCAGUCG...3′

B.

5′...CGUGAGC...3′

C.

5′...GCUGACG...3′

D.

5′...CGUCAGC...3′

E.

5′...CGUGAGC...3′

answer

tion, the

A.

is

mRNA direction

example

A.

Because

and

the

of is

the

all

nucleic

coding

transcription. coding

acids

strand

of

This

strand.

are

DNA

means

The

top

synthesized

must that

strand

the is

in

each

be

bottom

the

the

5′

to

oriented

5′

strand

template

in

3′ to

direc 3′,

i.e.,

in

this

strand.

Approach:



Cross



Identify



Find ber



out

any

option

the the

to

coding

option

strand,

U if

you

will

a

strand with

substitute

Alternatively,

with

a for

T.

of

DNA

sequence T,

you

prefer

get

the

if

identical

the to

direction the

of

coding

transcription. strand

(remem

necessary). to

same

from

find

the

complement

of

the

template

answer.

3

Immunology

PART

I

|

BIOCHEMISTRY

Biochemistry PRODUCTION

The

OF

structure

illustrated Medical Genetics

in

and

expression

Figure

I

of

3

4.

MESSENGER

a typical

The

prokaryotic

following

RNA

gene

events

occur

coding

during

for

the

a

protein

are

expression

of

this

gene:

1.

With

the

help

promoter the

the

plate

2.

base

core

ecule.

There

are

prokaryotic



Rho

independent

back

on

two

a

The

enzyme

the

promoter on

strands

factor

transcription

GC

rich

the

5′

is

to

of

the

tem

DNA

released

template

3′

as

it

reads

as

soon

as

strand

in

the

3′

to

direction.

transcription and

of

termination

release

the

signal,

completed

terminators

to that

polymerase

from and

the

of of

at

mRNA

commonly

at

has

paused

the

message

mol

found

in

message.

ribosome from

end

of

a the

Shine

and

the

RNA

folds

by

6–8

synthesized

U

RNA

template.

This

is

the

type

of moves site.

rho

factor.

toward

the

Rho

then

This RNA

displaces

RNA

RNA. simultaneously

in

(no

introns),

transcription

is

Dalgarno

sequence

synthesis

begins

continues

coding

terminus

DNA

termination

occur

before

newly

and

mRNA

Protein

the

at of

formed followed

participation RNA

prokaryotic

region

translates amino

end

the

the

newly closely

4.

formed

even the

of

from I 3

can

of

coding the

3′

the loop

requires

newly

translation

called

the

RNA

Figure

the

the

when

hairpin features

the

in

processing

a sequence

(UTR)

structural

termination

binds

no

occurs

of

dependent

is

a

shown

polymerase

protein

the

transcription

form two

terminator

Rho

The

two

Sigma

along

in

termination

dissociation

codon

mRNA

kinds

to

promote

ning

the

the

pair.

reaches

stop

These

translating

6.

will

itself

Transcription

to

base

moving

the

residues.

there

sequence.

orients

to sequences,

genes:

protein

5.

–35

binds

“consensus”

strand.

+1

eventually it

of

the

the

and

template

at

and

separates

continues

polymerase point

box)

and

two

initiated.

polymerase

which



the

synthesizing

RNA

TATA

recognizes contains

transcription

polymerase

begins is

direction,

4.

RNA

polymerase

promoter

(or for

of

Transcription

The

box

sequence

RNA

bacterial

site

The

transcription 3.

factor,

The

start

strand.

the

sigma

Pribnow

identifies

Behavioral Science/SocialSciences

of

region.

called

3

PROKARYOTIC

until

at

Because can

complete. in

the

bacteria. ribosomes

an

the AUG

ribosome

Ribosomes 5′

bind

untranslated codon

begin

region at

reaches

the

begin

a stop

region. message to

in carboxyl

the

5′

to

terminus.

3′

direction,

synthesizing

the

5′

CHAPTER

DNA

3

|

TRANSCRIPTION

AND

RNA

PROCESSING

Transcription TATA box –35

5

́

3

́

ATG

TGA Coding

–10

region 3 ́

5 ́ Promoter 5

́ Untranslated region

3

(UTR)

́ Untranslated region

(UTR)

+1 Transcription Transcription

terminates

mRNA Shine

AUG

Dalgarno

UGA Coding

sequence

GC

rich

and

loop

stem

region UUUUUU

5 ́

5

́

UTR

3

́

3 ́

UTR

Translation

H2N

I Figure 3 4.

Figure

The

mRNA That

protein.

The

example, related in

this

I 3 4. Expression Expression

produced

message.

the genes case

is,

by it

word

the

cistron

shown

operon)

grouped

another

ProteinProteinCoding

the

figure

single

above

gene

for

a

and

gene.

the

DNA

from

are

several

Gene Coding

is

for

In

transcribed

as

and

only

bacterial

messages.

genes

Gene

a monocistronic

codes

Some

polycistronic

in

information

a name

produce

together

contains

in

from

is

- COOH

Prokaryotic of of a a Prokaryotic

gene

is transcribed

lactose

- Protein

codes

a

single

operons these

one

unit.

for

(for

cases, The

several

mRNA different

proteins.

Promoter

5

́ UTR

Gene

1

AUG

Gene

UAA

AUG

2

Gene

3

AUG

UGA

́ UTR

UAG

5 ́ UTR

3 ́ Gene

Polycistronic

3

1

Gene

2

Gene

UTR

3

mRNA Shine

Dalgarno

Shine

Each

Dalgarno

gene

H2N–Protein–COOH

Shine

is

translated

Dalgarno

independently

H2N–Protein–COOH

H2N–Protein–COOH

12

Figure Figure

I 3 5. Polycistronic I 3 5. Polycistronic

3

Gene

Gene Region

Region Codes for

Codes Several

for Different

Several Proteins

Different

Proteins

3

Immunology

PART

I

|

BIOCHEMISTRY

Biochemistry PRODUCTION

OF

In

eukaryotes,

by

noncoding

most

Introns

nucleus. lated

in

coding as

genes

segments

nucleus.

Medical Genetics

EUKARYOTIC

are

during

eukaryotes,

all

cytoplasm.

The

protein

is

of

Both

removed

In

a

composed

(introns).

are

the for

MESSENGER

coding

exons

segments

and

processing

mRNA

RNA

introns of

and

illustrated

in

are

the

is monocistronic.

structure

RNA

The

I

3

of

6.

interrupted

transcribed

in

molecule

mature

transcription

Figure

(exons)

in

mRNA

a typical

the

the is trans

eukaryotic

Transcription

of

this

gene

gene

occurs

follows:

Behavioral Science/SocialSciences 1.

2.

With

the

of

and

eukaryotic

genes

(also

called

RNA

polymerase

initiate in

3.

help

recognizes

proteins to

3′

Hogness

to

sized

in

the

RNA

polymerase

These

signals

3′ II

are

the

RNA

not

ends

of

DNA

the

in

it

a short

primary

reaches

is

read

synthe

transcribed. a termination

signal.

eukaryotes.

Poly

Exon

is

transcript) are

to

strand

A

addition

signal

TAG

1

Exon

–25

box

region

template

introns

ATG

–70

of

TATA

DNA

Promoter

II region

the

over The

and when

understood

DNA

(the

exons

transcription

well

called

sequence.

product

Both

polymerase

promoter

box.

strands

the

RNA basal

sequences

CAAT

read

direction.

The

consensus

the

as

factors,

region.

the

separates

direction to

two

and

and

5′

transcription

promoter

has

box) II

5′

the

usually

transcription

the

called

binds

2

Intron

5 ́

3 ́

3 ́

5 ́ CAAT

TATA

box

box

5

́

Untranslated

region

3

(UTR)

́

Untranslated

region

(UTR)

+1

Transcription terminates

Transcription Poly AUG

UAG

Exon

1

A

addition

signal

Exon

2

Intron Pre

5

mRNA

́

3

5

́

UTR

3

FigureI 3 I 6.3

Figure

6.A AEukaryotic Eukaryotic

Transcription Transcription

́ UTR

́

HY

MY

Unit

Unit LY

Processing Pre The

of

Eukaryotic

Messenger primary

transcript

processing

inside

the

processing

steps

include

1.

A

7

High

RNA must

being

also

helps

protect

is

The the

form

extensive the

co

mature

and

post

mRNA

transcriptional MEDIUM

molecule.

LOW

added cap

mRNA

YIELD

to

the

structure chain

5′

end

serves from

while as

a

the ribosome

RNA

YIELD

molecule

FUNDAMENTALS binding

is site

and

FUN

degradation. REINFORCEMENT

38

ME

These

following:

cap

synthesized. to

to

the

methylguanosine

still

undergo

nucleus

H

Yield

R

CHAPTER

2.

A the

poly

tail

on

poly

A

poly

A

transport no

is attached

molecule

then The

A

poly

the

3′

side

polymerase tail

to A

to

3′

of adds

protects

the

the

the

end.

the

sequence

the

poly

message

cytoplasm.

A

few

In

this

A

tail

process,

an

AAUAAA

rapid

mRNAs

(poly

(about

against (for

endonuclease

200

As)

A

degradation example,

histone

|

TRANSCRIPTION

AND

RNA

PROCESSING

cuts

addition

to

3

signal),

the

new

3′

and

aids

in

end.

mRNAs)

its have

tails.

Poly AUG

Exon

A addition signal

UAG

1

Exon

2

Intron Pre

5

mRNA

́

3

5 ́ UTR

3

Capping

and

addition

AUG

Exon

Poly

(nucleus)

́

UTR

Poly

A

A addition signal

UAG

Exon

1

́

2

Cap Intron Me

hnRNA 5

́

AAAAAAAA Gppp

Poly 5 ́

UTR

5 ́ Splice

Splicing

by

3 ́

Splice

donor

acceptor

site

site

spliceosome

(snRNA)

A

tail

A

tail

3

́

3

́

3 ́ UTR

(nucleus)

3 ́ Excised (lariat)

Poly

intron

A addition

degraded in

signal

nucleus AUG

Exon

1

UAG

Exon

2

Cap Me mRNA

5

AAAAAAAA ́ Gppp

Poly

5 ́

UTR

3 Transport

to and

́ UTR

cytoplasm

translation

H2N–Protein–COOH

Figure

I

3

7.

Figure Processing

I 3 7. Eukaryotic Processing

Eukaryotic Pre

Pre mRNA mRNA

3

Immunology

PART

I

|

BIOCHEMISTRY

Biochemistry 3.

NOTE

Introns

are

(also Mutations

in

abnormal that Medical Genetics β

splice

proteins.

interfere globin

cases

of

β

For

with

mRNA

sites

are

proper

can

lead

example,

as

from

an

hnRNA

snRNP,

or

by

splicing,

snurp),

accomplished

which

are

by

complexes

of

spliceosomes

snRNA

and

to mutations

splicing

responsible

removed

known

of for

thalassemia.

protein.

The

ceptor)

ends

and some

of

All

is

intron.

The

Neighboring

of of

molecule

the

degraded.

region 4.

hnRNA

the

the

mature

cut

at

intron

splice

sites

exons

is excised are

at in

joined

the

5′

the

(donor)

form

together

to

of

and a

3′

lariat

assemble

(ac

structure

the

coding

mRNA.

intermediates

in

this

processing

pathway

are

collectively

known

as

hnRNA. Behavioral Science/SocialSciences

5.

The

mature

lated

to

mRNA form

molecule

ALTERNATIVE

PRE

SPLICING

mRNA

For

some

more

genes,

the of

splicing. in

this

unstimulated

primary

may

the

be

in

By

alternative

than

it

is

at

technique

discussed

because

genes

in

Chapter

they

should may

not change

be

memorized upon

́

A

T

by by

undergo

primary

giving

rise

alternative transcript

to

can

make

many of of

detected

Exon

a gene

specific

estimate be

of

tissue

estimate

current can

are

by

more the

number

human

of

genes

Northern

is

blot,

a

7.

pre

RNA

Exon

1

1

Intron Exon

2

transcript

1

Exon mRNA

Figure

2

Exon

2

Intron Exon

Splicing

Exon

variation

5

3 ́

́

Exon

1

Exon mRNA

Alternative

4

3

́

#2

#1

I 3 8.

3

3

Splicing #1

4

to

4

genes

current

the

splicing

variation

5 ́

or

UAA Intron

5

two alterna

troponin

AUG

more

research.

is trans

splicing.

the

organism

encode.

whereas

Primary

figures

of

an

to

as

immunoglobulins

or

tissues,

Alternative

NOTE

These

it

immunoglobulins

alternative

cell,

different

100,000,

20,000–25,000.

same

known

and

secreted

involves

splicing,

has

least

to

produce

is

bound

percentage

the

to

process

tropomyosin

opposed

a large

This

membrane

also

within

spliced

proteins only

of

as

occur

proteins

human

where

differently

gene.

proteins

synthesis

from

may

products.

different

same

B lymphocytes,

alternatively

protein

cytoplasm,

PRIMARY

is spliced

the

muscle

The

transcripts This

about

from

of

way.

stimulated

splicing.

the

EUKARYOTIC

transcript

protein

B lymphocytes,

antigen

The

OF

primary

a

Variants

produced

to

TRANSCRIPTS

variants

tive

is transported

a protein.

Splicing Produce

of Different

Eukaryotic Proteins

hnRNA

(Pre

mRNA)

3 #2

Exon

4

3

́

CHAPTER

RIBOSOMAL

RNA

CONSTRUCT

RIBOSOMES

Eukaryotic

ribosomal

a single

piece

rRNA,

and

separate

rRNA.

RNA

is

in

protein

Prokaryotic

nucleolus

by

cleaved

to

transcribes

assemble

proteins.

during

the

III

subunits

|

TRANSCRIPTION

in

to

RNA 28S

5S

the

the

from

subunits

whole

80S

Eukaryotic

a

rRNA are

60S

ribosome.

Ribosome

BRIDGE

80S

50S

Shiga 5S

RNA

TO

toxin

and 5.8S

(Shigella

Verotoxin,

a

RNA

28S

rRNA

in

16S

40S

RNA

18S

RNA

subunit

are

single

adenine prevents

ribosome,

I 3

The

large

The

complete

and

The I 3 Composition 9. The Composition

small

by

both

prokaryotic

size

and

TRANSFER

ribosome

behavior

of

shape,

TRANSLATION

There

are

activated these

tRNAs

enter

the

many

different acid in

eukaryotic

Although

all

tRNAs

structural

features

the

distinguish

tRNAs.

making cells

are

they

combine

the

same among

(Note:

are

and The

tRNA

transcribed

shape

their

only The

the

the

RNA

The

A subunits

glycosylases residue aminoacyl

halting

protein

that from tRNA

of

these

remove

the

28S binding

a rRNA. to

the

synthesis.

a

function

in

ACIDS

one

type

genes

polymerase

appropriate shown

of

are

are

AMINO

carries

RNA

inactivate

additive.)

translation. by

with

coli), subunit

respectively.

S values

They not

60S

toxin

RibosomesFigure

30S,

ACTIVATED

during

general

50S

ultracentrifuge.

numbers

Each

proteins

are

particle. an

CARRIES

where have

a 70S in

therefore

and andEukaryotic Eukaryotic Ribosomes

subunits

is

specific for

ofProkaryotic Prokaryotic

particles

(tRNA)

amino

cytoplasm

the

and

RNA

FOR

of

ribosomal

prokaryotic

determined of

9.

E. the

ribosome.

toxins

This

Figure

like

RNA eukaryotic

30S

dysenteriae) Shiga

(enterohemorrhagic 28S

subunit

MICROBIOLOGY

RNA

60S subunit

23S

PROCESSING

18S

unit

as

70S

5S

RNA

I as

rRNA,

rRNA

nucleolus ribosomal

form

polymerase

yield

the the

Eukaryotic

synthesis

Ribosome

subunit

AND

TO

subsequently

polymerase

ribosomal

join

USED

transcribed

which

ribosomal

with

They

IS

is

RNA,

The

combine 40S.

RNA 45S

5.8S gene.

pieces and

of

(rRNA)

3

III. amino

Figure

I

of

encoding The

tRNAs

acids. 3

10,

small

them.

4

Immunology

PART

I

|

BIOCHEMISTRY

Biochemistry 3

́ end

A

OH

Activated acid

C

to Medical Genetics

amino is

3

attached

́ OH.

C 5

́ end

A

C

A

G

C

C

G

G

C

G

C

G

C

G

C

P

Behavioral Science/SocialSciences

A C

G

G

C

A

U

C

U C

A

G A

C

C

G

G

A

G G

U

U

C

G

G T

C G

G

A G

C

U

C

C

ψ

U

D

G G A

U C

G

G

C

G

C

G

C

A

G

A

Cm U

A

C

U Anticodon

A

sequence

pairs

Figure Figure

RNA

Editing

RNA

editing

is

nucleotide RNA

base RNA

tRNA

molecules

important

4

In codon,

the thus

RNA

some

RNA

cells

discrete its

events

(such

as

has

been

editing

mRNA.

(tRNA)

make after

editing

in

(tRNA)

RNA

molecule

(CAU)

codon

may

adenine

changes

gene

has

to

been

include

insertion,

deamination)

observed

in

specific

transcribed

within

some

mRNA,

by deletion,

the rRNA,

and

humans.

example is

Transfer

Transfer

nucleotides RNA

in

B100

intestines.

of

molecule.

Apoprotein

which a

10.

10.

Posttranscription

alterations

edited

a stop

by within

polymerase.

and

An

a process

sequences

I 3

I 3

with

is

cytosine

expressed

intestines, producing

to in

the

the

uracil liver,

mRNA the

shorter

deamination and

is

in

apoprotein

edited apoprotein

from

the

B48 a CAA B48

form.

apoprotein is

B gene.

expressed

sequence

in to

be

the UAA,

CHAPTER

Recall

Question

Which

eukaryotic

A.

mRNA

B.

rRNA

C.

tRNA

RNA

contains

I 3

2.

Important

Points

About

regions

Transcription

and

May

be

RNA

Core

of

transcription

continuous

little

coding

spacer

between

Initiation

regions

(noncoding)

DNA

genes

enzyme:

α2ββ′

Promoter

(–10)

TATAAT

initiation

recognize

Template

of

Relationship transcript

transcription

of to

RNA

DNA

and

Posttranscriptional processing (pre

subunit

(–35)

to

have

Large

spacer

and

introns

(noncoding)

DNA

RNA

polymerase

I:

RNA

polymerase

II:

mRNA;

RNA

polymerase

III:

tRNA,

(–25)

between

rRNA

TATA

snRNA 5S

and

factors

read strand

3′

to

5′

5′;

to

mRNA

3′;

and

loop

+

UUUUU

Stem

and

loop

+

rho

is

antiparallel

(except

U

5′ begins

to

at

Not

RNA

(–70)

(TFIID)

CAAT bind

3′;

+1

gene

sequence

specified

from

base

well

characterized

factor

and

substitutes

synthesized

transcription

complementary for

T)

to

to DNA

DNA

coding

hnRNA

template

strand;

RNA

is

identical

strand

In

nucleus:

5′

cap

(7

3′

tail

(poly

MeG)

mRNA)

Removal •

Ribosomes

70S

(30S

rRNA

and

Cloverleaf

and

50S)

protein

rRNA

secondary



Acceptor



Anticodon

80S

A of

sequence)

introns

from

Alternative of

tRNA

exons

promoter

None of

Genes

Promoter

required

Stem

RNA

monocistronic

promoter

coding

Termination

Always

Transcription

Sigma

synthesis

PROCESSING

genes

sequence

mRNA

RNA

Processing

polycistronic are

Very

polymerase

AND

Eukaryotic

Genes

RNA

TRANSCRIPTION

A

Prokaryotic

Gene

|

UTR?

Answer:

Table

3

splicing

protein

(40S and

pre yields

RNA variants

product

and

60S)

protein

structure

arm arm;

(CCA)

carries

anticodon

amino complementary

acid and

antiparallel

to

codon

in

mRNA

4

Immunology

PART

I

|

BIOCHEMISTRY

Biochemistry Review

Medical Genetics

Questions

Select

the

ONE

1.

The

base

Behavioral Science/SocialSciences

2.

answer.

sequence

of

CAGCGC.

The

tain

sequence?

which

A.

GCGCTG

B.

CUGCGC

C.

GCGCUG

D.

CAGCGC

E.

GUCGCG

A

gene

1,000 of

Items

best

200

bp.

start

AUG

A.

1,750

B.

750

C.

650

D.

450

E.

150

3–5:

mRNA

encodes bps,

Identify

a In

codons

the

final

the

150 region

processed

to

the

the

cytochrome

β5

transcription

amino

acids.

of

bp,

100

mRNA,

final

nuclear

in

upon

with

untranslated

codon

58

produced

a protein 5′

57

There and

how

termination

of

many

this

is

a 3′

reductase gene

one

intron

nucleotides

lie

location.

A C

4

Transcription

4.

Euchromatin

5.

Polyadenylation

of

genes

by

of

mRNA

pre

RNA

polymerase

by

poly

1

A

polymerase

is

con

of region

ED

3.

will

untranslated

codon?

B

gene

from

the

CHAPTER

3

|

TRANSCRIPTION

AND

RNA

PROCESSING

Answers

1.

Answer: the

D.

Since

mRNA

means

no

U

in

2.

Answer:

D.

Only

3.

Answer:

B.

rRNA

4.

Answer:

A.

Less

areas

5.

Answer: Generally

are

the

sequence

sequence

must

the

in be

the

stem

identical

represents

(except

U

the for

T).

coding No

strand,

T

in

the

3

×

150

DNA

mRNA.

the

coding

genes

condensed

region

are

remains

to

transcribed

by

chromatin,

lighter

this

be

calculated:

enzyme

areas

in

in

the

the

=

450.

nucleolus.

nucleus.

Darker

heterochromatin.

A.

Polyadenylation associated

of with

active

pre gene

mRNA expression

occurs

in in

the

nucleoplasm.

euchromatin.

4

The

Genetic

Code,

Mutations, 4#

and

LEARNING

Translation

OBJECTIVES



Demonstrate



Solve



Interpret by

understanding

problems

of

concerning

scenarios

the

genetic

code

mutations

about

amino

acid

activation

and

codon

translation

tRNAs



Demonstrate



Explain



Interpret



Answer

understanding

of

information

related

scenarios

to

about

questions

translation

inhibitors

protein

about

how

(protein

of

folding

translation

synthesis)

protein

and

synthesis

subunit

occurs

on

assembly

the

rough

endoplasmic

reticulum



Demonstrate

understanding

of

co

and

posttranslational

covalent

modifications



Solve

problems

concerning

posttranslational

modifications

of

collagen

TRANSLATION The

second

stage

messenger

RNA

genetic in

code

DNA

Each

(or

is

is

into as

the

the

translating

amino

acid

relationship

transcripts) by

the

one

or

of the

sequence more

nucleotide

sequence

between

and

specified

the

sequence a protein.

sequence

of

amino

nucleotide

a

The of

acids

triplets

of

nucleotides in

a protein.

(codons)

in

DNA.

translation, for

mRNA.

tRNAs

the

mRNA

each

amino

before

amino serve

acids peptide

protein

interactions

they

RNA

acid

codons

for

expression

defined

its

During

of

gene

molecule

is

amino

the

in

protein prepare

they bond

and

rRNA between

synthesis. to

assume

acts

acid as

as

in

the

adapter

each

specify,

thus

Many their

copy

as tRNA,

been

that

couple

aligning

proteins ultimate

molecular the

gene

place

the

and posttranslational

in

the

in

which

the

from

DNA

codons

in

the on

machines

enzymes,

undergo roles

the

transcribed

them takes

the

of

have

Translation

serve

mRNA,

working

protein

molecules

formation. that

a

in

appropriate

protein

with sequence

ribosomes,

complexes

coordinating the

to

mRNA

the factors

modifications

required as

cell.

4

Immunology

PART

I

|

BIOCHEMISTRY

Biochemistry THE

GENETIC

Most es.

CODE

genetic

code

Important

Medical Genetics



of

codon

written 61

designate

features

Each



tables

consists

in

the

codons (or

genetic

of

5′

to

code

codons

the

3′

for

the

3

code

bases

for

amino

acids

as

mRNA

sequenc

include:

(triplet).

There

are

64

codons.

They

are

all

direction.

amino

nonsense

codons

acids.

codons)

The that

other

3

terminate

(UAA,

UGA,

UAG)

are

stop

translation.

Behavioral Science/SocialSciences •

There

is

Protein

one

start

codon

synthesis

formylmethionine •

The

(initiation

begins

with

(fMet)

code

is

unambiguous.

is

degenerate.

codon),

AUG,

methionine

in

coding

(Met)

in

for

methionine.

eukaryotes,

and

prokaryotes. Each

codon

specifies

no

more

than

one

amino

acid. •

The



code

acid.

All

one

codon.

For

those

codon •

The to



this code

between

in is

more

same.

The

(the

same

codon

and

than base

in

can

specify

tryptophan

all

one

in

codon,

the

a

(Trp),

third

the

first

position

organisms).

Some

commaless on codons

(contiguous). an

2

minor

than

bases

in

varies. exceptions

There

are

no

spacers

or

“commas”

mRNA. on

a

First

message

Second

are

nonoverlapping.

Third

Position

Position U

́ End)

UUU U

}

UUC UUA UUG

}

C

UCU

Phe

UCC

}

CUA

UCA Leu

A

Leu

}

Pro

CCA

ACC

Thr

Met

GUU }

Val

Trp

G

AAA

GAC

GCG

GAA

U

CGU

His

CGC

}

CGA }

}

Gln

}

}

AGU

Asn

AGA Lys

AGG

Asp

GAG

I 4 1. The Genetic Figure I 4 1. The Genetic

Code Code

GGG

A

U

}

Ser

}

Arg

C A G

U

GGU GGA

Glu

C

G

AGC

GGC }

Arg

CGG

AAC

GAU Ala

}

Stop

CAA

GCC

}

}

CAC

GCU

GUG

Figure

A

UGG

AAG

GCA

C

Stop

ACG

GUA

Cys

UGA

AAU }

U }

UAA

CAG

ACU lle

UGU

Tyr

(3

UGC

CAU }

ACA

AUG

G

CCC

}

G

UAC UAG

CCG

AUA

GUC

Ser

UCG

CUG

AUC

UAU }

CCU

CUC

AUU

A

C

CUU

48

amino

more

often

Position (5

single

have

mitochondria.

codons

Neighboring

one

Met

having

the

universal

occur

than

except

acids

usually is

More

acids,

amino are

code

The



amino

}

C Gly

A

́

End)

the

CHAPTER

4

|

THE

GENETIC

CODE,

MUTATIONS,

AND

TRANSLATION

MUTATIONS

A

mutation

is any

organism.

This

sequence

of

altered

Mutations

enzyme

activity,

antigenicity,

A



of

with

a different

base

pair

becomes

becomes

a T are

gene’s

often

protein

the

A

code

table

can

also

in acid

of

the

an

base

sequence

cause

of

changes

a

in

morphology,

alteration

a

base

or

point

purine

pair.

mutation.

pyrimidine For

base

example,

an

A

T

replaces

pair.

For

the

effect

a

purine

pyrimidine

example,

an

A

T

base

base

pair

pair. to

change

in

amino

replaces

that

according This

changes

the

susceptibility,

base

base

base

classified product.

genetic

G

sequence

pair.

mutation

a C

by

in

pyrimidine

purine

or

base

cells.

that

base

a point

a pyrimidine

reflected

a single

mutation

C

DNA

antibiotic

is

G

the

They

of

purine

is

with

Mutations

mutation

a

transversion

be

diseases.

properties

a point

pair

pair

using

other

in

changes

requirements,

many

is

can by

genetic

nutritional

transition

A

sequence

cause

type

change

sometimes,

can

common



the

and,

and

very

heritable

DNA

mRNA,

protein.

A

permanent,

in

they

protein

conjunction

have

structure

with

the

on

can

base

the be

structure

of

predicted

sequence

of

DNA

A

T

or

mRNA.

Normal

A

T

G

G

Met

Silent

A

C

A

A

T

Ala

T

G

G

C

T

CGT

Ile

A

A

T

T

Arg

T

T

T

T

Phe

CGT

T

T

A

CCT

Leu

T

T

T

Pro

TG

A

G

Ile

CCT

A

G

G

Gly

T

A

G

G

DNA

Amino

G

coding

strand

coding

strand

coding

strand

coding

strand

Acid

DNA

Mutation Met

Missense

A

Ala

T

G

G

C

Ile

A

A

T

Arg

Phe

TCGT

T

Leu

T

T

TC

Pro

A

Ile

CCT

A

Gly

T

A

G

G

Amino

G

Acid

DNA

Mutation Met

Nonsense

Ala

ATG

G

C

Ile

A

A

T

Arg

T

CG

Phe

T

T

Ser

T

T

T

Pro

G

A

Ile

CCT

A

Gly

T

A

G

G

Amino

G

Acid

DNA

Mutation Met

Ala

Ile

Arg

Phe

Stop

Amino

Acid

Frameshift A

Mutation (1bp

deletion)

T

G

G

Met

C

A

AT

Ala

T

CGT

Ile

Arg

Figure

I 4 Figure

Large Large

Segment

somes

in is

of

DNA

meiosis. part

cells event,

can

be

Crossover

a normal

reproductive crossover

T

T

T

Phe

2.

A

C

CT

Tyr

Some

I 4 2. Some

A

A

G

Leu

Stop

Types

of Mutations

Common Common

T

G

G

coding

DNA Amino

Types

of

Mutations

in

strand

Acid

DNA

in DNA

Deletions

segments

crossover

T

(egg the

of

or

meiosis

and homologous

deleted

sperm),

from

a chromosome

recombination I that a maternal

during

between

generates largely

genetic beneficial

and

paternal

an

homologous diversity result.

In

chromosomes

unequal chromo

in a normal exchange

4

Immunology

PART

I

|

BIOCHEMISTRY

Biochemistry equivalent

segments,

maternal

and

one.

On

loses

rare

some

and

although

paternal

alleles,

occasions,

a

of

its

the no

crossover

genetic

resultant

genetic

chromosomes

information

can

be

has

unequal

been

and

one

of

are

mosaics

lost

from

the

of either

two

homologs

information.

Medical Genetics

HY α

thalassemia

is

crossover chat

has

a

like

known

deleted

(intellectual

kitten

well one

example

or

more

disability, cry)

of

α

a

globin

from

from

wide

a terminal

disease

genes

microcephaly,

results

genetic

set

deletion

in

unequal

chromosome

eyes,

of

which

and

the

16.

Cri MY du

a characteristic

short

arm

of

LY

chromosome

5.

Behavioral Science/SocialSciences

Mutations

in

Mutations

in

Splice

splice

sites

posttranscriptional somes

High

Sites affect

the

processing.

If

accuracy

of

a splice

intron

site

is

MEDIUM from hnRNA YIELD during

removal

lost

through

mutation,

Delete

LOW



Leave



Use

nucleotides

from

the

adjacent

YIELD

exon FUNDAMENTALS

nucleotides the

from

next

the

Mutations

in

including

of normal

intron

upstream

processed splice

β

the

sites

have

now

is a β

are

two

globin

that

of

and

frameshift

(more

A

9

his

an

month

severely

due

to

are

the

against

red

marrow bones

5

the

face skull.

from

common

treatment

a serious

consequence.

and is

A

is

there

large

number

deletions,

mutations

β

globin

mRNA

not

encoding

information

site

mutations

at

intron

of

involving

involve

splice

was

the

1 create

a new

splice

to

hospital

by

brought

listless,

and was

unusual

in and red

due

to

of

with

the

the

in role

face weak every

that

large

areas. 6

The

abnormally transfusions

features

cells

irritable. and

Mediterranean

blood

the

frequently enlarged

glucose

bloodstream.

are

diseases,

thalassemia,

defects

gene as

spleen

deformities

blood

globin

β

globin. gene

mutations

within

had

is

legs

α

Other

such

primarily

the

different

In

with

translational

β

the

anemia of

many

hemoglobin.

including

descent

that

the

cell

bone arms

exon

Sachs.

and

pale,

Splenomegaly

produces of

REINFORCEMENT deleting an

site,

site,

mRNA.

Greek

abnormality

cells

the

became

found

sickle

malaria.

damaged

of

out),

His

Tay

compared

mutations

noted

in

to

deficiency,

of

of he

anemic.

similar

splice

THALASSEMIA

chain

long

infant

deformities

thalassemias

FUN

mRNA

in

and

described,

spliced

Also,

physician

was

that,

be

because

attending

been

abnormally

old

parents

beta

70%

must

in

processed

documented

mutations.

sequences.

resulting

downstream

process,

than

eventually

or

disease,

protein

have

transcriptional

consensus

the

globin

the

processing and

for β

mutations

slow

nonsense

β

genes

deficiency

the

been

Gaucher

β

There

in

mRNA

thalassemia,

ME

spliceo

may:



H

Yield

It

phosphate β

the

believed

may

spleen

in

activity other

and 2–3

is

dehydrogenase

excessive and

infant

cheekbones

thalassemia

of

The the

areas.

fracture weeks,

clearing of The

easily. but

protect

iron

the

bone long The

overload

most is

RE

MY

4 LY|

CHAPTER

Trinucleotide The

Repeat

mutant

alleles

syndrome, the

and

number

repeats

in

certain

diseases,

myotonic

of

increases

and

decreasing

such

dystrophy,

tandem

often

High

Expansion

copies

of

with

as

differ a

their

trinucleotide.

successive

In

normal these

generations

and

YIELD X only

the

LOW YIELD number

correlates

with

GENETIC

HIGH

of

disorder,

has

decade

4.

example, the

in

the

coding

age

normal

region.

repeats.

The

proteins

normal

The

a

the

long

contains disease

A

clinical

disorders

is neurodegeneration

5

makes

have

the

30–60

of

or

the

adjacent

proteins

expansion

region

or

of in

an

the

trinucleotide

untranslated

region

of

in the

the

followed

allele

can

I 4

1.

Two

Translation

Classes

repeat

(polyglutamine

of

Trinucleotide

be

in

a

by

of

(CAG)n

muscular

Fragile

atrophy:

repeat

until

death

impaired the

gait,

dystonia,

memory,

first

signs,

chorea

(loss

dementia,

of

and

onset

most

(age

frequently

1

acid)

codon);

missense,

1

segment

Met)

or

2

often

differ

at

base

3

nonsense

nucleotides;

not

multiple

of

3)

deletion

Mutation

in

splice

site

Trinucleotide

repeat

expansion

Amino

acid

Aminoacyl

tRNA

synthetase:

two

high

energy

bonds

(ATP)

to

link

amino

acid

to

cap

on

tRNA

activation

Translation:

30S

Initiation

sequence

subunit

binds on

fMet–tRNAi

GTP

Translation:

Charged

Elongation

A

to

Shine

Dalgarno

40S

binds

to

P

site

Peptide bonds

associates

Met–tRNAi

required

site

subunit

with

5′

mRNA

mRNA

GTP

aminoacyl–tRNA

binds

to

binds

to

P

site

required

Charged

aminoacyl–tRNA

binds

to

A

site

(GTP)

(GTP)

bond from

forms amino

(two acid

high

energy

activation)

28S

rRNA

is

removing

cut

an

by

Shiga

adenine

and

Shiga

residue.

like

toxins

Prevents

protein

synthesis.

Peptide

bond

amino

Peptidyl

transferase

Translocation:

(50S

GTP

subunit)

required

acid

Peptidyl

forms

transferase

Translocation:

eEF

(two

2

high

energy

bonds

from

activation)

(60S

GTP

required

by

Diphtheria

inhibited

subunit)

and

Pseudomonas

toxins

Termination

Protein

Release

targeting

of

protein;

protein

synthesized

N

to

C

Secreted

or

membrane

hydrophobic Lysosomal

Other disease

important

Scurvy Menkes

by

N

terminal

sequence

enzymes:

mannose I cell

proteins:

signal

phosphorylation

of

phosphotransferase

in

Golgi

disease

(prolyl Disease

hydroxylase, (Cu

Vit deficiency,

C) lysyl

oxidase)

associations

6

Immunology

PART

I

|

BIOCHEMISTRY

Biochemistry Review

Questions

Select

Medical Genetics

the

1.

In

Behavioral Science/SocialSciences

in most

the

best

amino

acid

is

A.

AAC

B.

UAG

C.

UCC

D.

AUG

E.

UAU

2

code acid

and

human

ATGCAA.

. .→

ATGTAA

B.

ATGAAA.

. .→

GTGAAA

C.

TATAAG.

D.

CTTAAG.

. .→

GTTAAG

E.

ATGAAT.

. .→

ATGCAT

options

. .→

above

boldface

nuclear

is

A.

UAC.

DNA,

one

Another

closely

For

mutations

each

related

mutation

3.

Mutation

decreasing

4.

Accumulation

affected

by

this

in

in

of

same

amino

2.

can

Which

of

the

globin steps

mRNA

B.

Attachment

of

the

to

the

endoplasmic

C.

Met

D.

Translocation

E.

Attachment

tRNAmet

of of

to

mRNA

RNA

swab

paroxysmal

coughing was

ribosome

binding

nasopharyngeal

most

A.

erythromycin

B.

tetracycline

C.

chloramphenicol

D.

rifamycin

E.

levofloxacin

on

obtained

P

the

synthesis

by

is most

directly

the

with

rhinitis

to of

ribosome

with

II

to

hospital 70S

the

a 4

positive the

the

reticulum

site

from

tested

admitted

treated

the

polymerase

translocation likely

choose

mechanism?

pre

He

indicated

below,

regulate

to

the

changes

following

spliceosomes

pertussis.

base

transcription

of

was

specifying this

above.

Attachment

inhibits

codons

questions

options

reticulocytes

eIF

with the

A.

and

the

specifying

DNA in

the

initiation

heme

inactivating

the

described

change

the

of

in

mutation

sequence

Nonsense

A

of

codon

TCTAAG

represent

type.

2.

5.

of

tyrosine

3

indirectly

6

answer.

genetic

the

Items

The

ONE

ribosomes

promoter

month

upon for

old culture

therapy on

infant for with

the

mRNA.

Bordetella an

antibiotic This

that patient

CHAPTER

6.

A

25

month

old

hyperplasia symptoms

of

show

lowing

Golgi

B.

Lysosomal

C.

Endoplasmic

D.

Cytoplasmic

E.

Lysosomal

α

is

B.

First

C.

Middle

D.

Last

E.

3′

9.

from

recessive of

a 12 of

base

Factor

of

immunoreactive V

serum,

and of

the

fol

observations?

peptidase

the

bleeding Factor

pair

deletion

by

hepatocytes

V

Factor

gene

would

this

V

disorder

V

blood

in

the

Factor

and

results

antigen

in

mutation

characterized

coagulation V

gene

that

an

abnormal

in

the

most

by protein.

cytoplasm.

likely

be

In

which

located?

region

region

most in

abundant

many

tissues,

B.

proline

C.

hydroxyproline

D.

cysteine

E.

lysine

month

infant

is

level

prominent. to

these

A.

Lysyl

B.

Prolyl

C.

γ Glutamyl

D.

Phosphotransferase

E.

α

1,4

If

could

is

hematoma.

copper

protein

tissues.

one

old

subdural

relates

and

her

A

autosomal

Untranslated

glycine

delay

progressive

Which

these

TRANSLATION

phosphorylase

Factor

the

serum

with

AND

exon

several

6

consistent

MUTATIONS,

intron

amounts

and

cytoplasm.

CODE,

exon

A.

A

an

Untranslated

Collagen,

of

1,4

secretion

the

the

signal

concentration

arises the

5′

in

in

GENETIC

gingival

hepatomegaly, elevated

THE

glucosidase

hexosaminidase

accumulation

ing

1,4

plasma

A.

are

reticulum–associated

Parahemophilia

of

inclusions most

and multiple,

contractures,

enzymes

is

features

developing

joint

lysosomal dense

facial

began

|

phosphotransferase

α

Deficiency

coarse

age,

disability, of

associated

a reduced

has of

deficiencies

A.

impairs

girl

months

phase

enzyme

region

2

Levels

fibroblasts

8.

at

intellectual

cardiomegaly.

7.

Caucasian

and,

4

seen The

is

5.5

nM

A

deficiency

in

one

the

measure

their

in

emergency

the

child’s (normal of

human

wished

hair for which

to

body,

compare

content

thin,

age, enzyme

present

in

collagen

vary

content

of

room is

is the

with

colorless, 11–12

nM). activity

a

fractured

and

tangled.

rib His

Developmental most

closely

symptoms?

oxidase hydroxylase carboxylase in

Golgi

glucosidase

6

Immunology

PART

I

|

BIOCHEMISTRY

Biochemistry 10.

Respiratory ed

tract

with

the

toxin

infections

secretion

most

caused

of

likely

have

of

by

exotoxin

on

A

eukaryotic

nitric

Pseudomonas

by

this

aeruginosa

organism.

are

What

effect

associat will

this

cells?

A.

Stimulation

oxide

(NO)

B.

ADP

ribosylation

of

a

C.

ADP

ribosylation

of

eEF

2

D.

ADP

ribosylation

of

a

protein

E.

Stimulation

synthesis

Medical Genetics

Behavioral Science/SocialSciences

11.

A

4

year

upper oral

of

old

tide

CF

Prior as

a

sequences

alleles

Codon

Normal

Mutant

are

with

and

therapy.

causing

release

cystic

infection

ciprofloxacin

gene

3 of

compared

Number

Gene

Gene

fibrosis

with is

to

testing pair

codons

a of

deletion 506–511

is

seen

by

his

physician

aeruginosa.

CF the in

in

center

as

patient

identified

exon

this

10

of

region

of

He

a potential

the the

for started

gene.

normal

The

506

507

508

509

510

511

ATC

ATC

TTT

GGT

GTT

TCC

ATC

AT•

••T

GGT

GTT

TCC

3

base

for

mutation

and

an on

candidate the

CF

is

below.

deletion

6

(CF)

Pseudomonas

referred

genetic base

protein

Gi

histamine

toddler

respiratory

Gs

nucleo mutant

CHAPTER

What

effect

protein

will

this

encoded

patient’s

by

the

mutation

CF

U

C

} Leu

}

GUU GUC GUA GUG

G

Deletion

UCU UCC UCA UCG

Phe

Leu

AUU } AUC AUA AUG

A

A.

}

CUU CUC CUA CUG

C

lle Met

}

of

on

Val

a

the

amino

acid

|

sequence

of

THE

GENETIC

CODE,

MUTATIONS,

AND

TRANSLATION

the

gene?

U

UUU UUC UUA UUG

have

4

}

A

Ser

CCU CCC } CCA CCG

Pro

ACU ACC ACA ACG

}

Thr

GCU GCC GCA GCG

}

Ala

phenylalanine

G

UAU } Tyr UAC UAA } Stop UAG

UGU UGC UGA UGG

CAU } CAC CAA CAG }

Gln

CGU CGC CGA CGG

AAU } Asn AAC AAA } Lys AAG

AGU AGC AGA AGG

GAU } GAC GAA } GAG

GGU GGC GGA GGG

His

Asp Glu

residue

with

U

} Cys

C

Stop Trp

A G

U }

C

Arg

A G

} Ser

U

} Arg

A

C G

U }

C

Gly

A G

no

change

in

C

terminal

sequence B.

Deletion

C.

Deletion

of

a leucine

residue

of

a

phenylalanine

of

a

leucine

causing

a change

in

causing

a

residue

the

C

terminal

change

in

sequence the

C

terminal

sequence D.

12.

A

Deletion

10

year

tance

old

to

boy

infection,

genetic

deficiency

a single

base

well

as

deletion

type

of

gene

A.

Frameshift

B.

In

C.

Missense

D.

Nonsense

E.

Splice

residue

with

severe

and

impaired

of

the

of

a

at 45

base was

no

progressive

change

skin

cognitive

enzyme

substitution

mutation

with

exon most

3’

end (exon

likely

C

intron 7)

inherited

in

sequence

decreased

has

been

Mutation of

terminal

ulceration,

ability

prolidase.

the

in

analysis 6

the

of

the

this

has

mutant

prolidase by

resis

diagnosed

cDNA.

with

a

identified allele

as

Which

boy?

mutation

frame

mutation mutation mutation site

mutation

6

Immunology

PART

I

|

BIOCHEMISTRY

Biochemistry Answers

1.

Answer:

E.

position) acceptable,

Medical Genetics

2.

3.

A. in

C.

is

is

a

stop

the

third

nucleotide

amino

acid.

position

Option

B

(wobble

would

be

codon.

contains

transcription

the

TAA,

eIF

A.

promoter

distractor

2

B.

designates

of

The

a The

placement

Answer:

at same

now

translation.

phase

5.

it

sequence

The

C.

eukaryotic

differ the

which

will

be

transcribed

to

mRNA.

choose

Answer:

UAU specify

that

The

the

Answer: Don’t

4.

and

often except

Answer: UAA

Behavioral Science/SocialSciences

UAC

and

TATA

protein

only

tRNA

Erythromycin

is

the

is

of

listed

that

in

the

P

antibiotic

been

not

factor

event

initiator

has

question

changed

about

the

to

TCTA.

translation.

initiation

phase

would

occur

in

during

this

site.

of

choice

for

pertussis.

It

inhibits

translocation.

6.

Answer:

A.

lysosomal

Characteristic

symptoms

enzymes

into

of

serum,

which

I

cell

disease.

would

not

Note

be

seen

release in

the

of other

deficiencies.

7.

Answer: tion

B. of

nascent the

N

to

terminal

Answer:

by

C. rich

the

V

acid

the

first

glycine,

signal

a

defect

in

sequence

of

is many

and

a

reticulum.

exon

Hydroxyproline in

secretion

suggest

endoplasmic

amino

encoded

is also

factor antigen

protein

and

8.

Decreased

cytoplasmic

the

accumula

translocation

This

implies

required

of a

for

the

mutation

targeting

in to

the

ER

gene.

found

other

corresponding the

uniquely

in

proteins

collagen.

contain

Although

significant

collagen

amounts

of

glycine.

9.

Answer: port in

A. is

10.

11.

Answer:

12.

7

weak,

C.

Answer: C

ile

(the

A.

in

E.

poorly

and

of

base an

cellular

His

diphtheria

deficiency.

fragile

connective

copper

bones

and

trans

Lysyl

oxidase

blood

vessels

tissue.

toxins

inhibit

eEF

2,

the

translo

translation.

of

CTT are is

results

exon

only

unaltered

in

the

because

loss ATC

of

phe

and

508; ATT

ile both

507

and

code

for

unchanged).

substitution entire

which copper

copper.

crosslinked

eukaryotic

sequence

in

a functional

requires

sequence

A

disease,

produces

Deletion

coding

deletion

Menkes

Pseudomonas

terminal

Answer: the

and

factor

the

has

metabolism

from

cation

child

abnormal

collagen

result

The

at is

an

indicative

intron

exon of

a

junction, splice

site

which mutation.

leads

to

Regulation

of

Eukaryotic 5#

Gene

LEARNING



Expression

OBJECTIVES

Demonstrate

understanding

of

regulation

of

eukaryotic

gene

expression



Know

concept

GENETIC of

integrity

of

through

gene

a

expression

cell.

rate

of

or

mechanisms,

specific

of

the

(enhancer

In

TFIID

in

process.

or

addition

transcription

are

feature

of

factors

to

the

regulatory

expression

of

important

ones

a

gene

to

can

occur

involve

regulating RNA

proteins specific

eukaryotic

help DNA

gene

expression

functional

proteins,

repressor bind

with

gene

the

transcription and

proteins

and

maintaining

the basic

associated

important,

in

the

activator

elements)

mechanisms

many

eukaryotes

These

silencer

essential

decreasing but

transcription. and

rate

Other

of

is an

Increasing

various

polymerase the

examples

REGULATION

Regulation

the

and

control

sequences

regions.

is controlled

at

multiple

levels.

REGULATION In

OF

eukaryotic

cells,

typically

requires

accessible

to

for

gene





DNA

RNA

Histone

in

and

in other

that

of

bind

structures,

order

to

proteins

aspects

factors

EXPRESSION

chromatin

remodeling

Important

histone

GENE

packaged

polymerase

Transcription as

is

chromatin

expression.

such

EUKARYOTIC

to

make

(transcription

chromatin

the

and the

gene

desired factors)

remodeling

DNA

and

expression gene

region required

include:

recruit

other

coactivators

acetylases

acetylases

(favor

gene

expression)

and

deacetylases

(favor

inactive

chromatin) •

Certain

lysyl

charge •

A

and

chromatin

reconfigures •

Additional RNA

residues weakening

in the

remodeling the

DNA

transcription

to

the

histones

are

interaction

acetylated

with

engine

that

binds

expose

the

promoter

factors

bind

in

to

the

decreasing

the

positive

DNA. acetylated

lysyl

residues

and

region. promoter

region

and

recruit

polymerase.

7

Immunology

PART

I

|

BIOCHEMISTRY

Biochemistry

H3

H3

H2A

H2A

H2B

Medical Genetics

H2B

Transcription

H4

H4

factor

Behavioral Science/SocialSciences

Transcription factor

Co

activator

histone acetylase

Transcription factor

Histone acetylase

Acetylated

lysyl

residues

in

histone

Chromatin

remodeling

residues for

and

engine

reconfigures

additional

transcription

Figure I 5 1.

the

transcription

maintaining the

cell.

The

general

are

case

specific together

when

the

such are

as

DNA

transcription to

form

1.

TFIID,

assembled

and

sites others).

Remodeling

basal of

lysyl expose

(low

the in

level)

protein this

transcription encoded

complex

occurs,

by

are

this

gene

referred

to

expression

of

hormones,

growth

sequences factors.

an

gene

of

that

allows

should

factors,

referred Several

enhancer

the

to these

as

be

response

response

control

increased

intracellular

of

in

elements elements

gene

response

conditions. that may

be

expression

In bind

by

grouped multiple

signals.

–75

GRE

CRE

–25

CCAAT

ERE

UPE

box

GC

Rich

+1

TATA

Transcribed

box

regio

1000 Enhancer

Figure

72

in

as

factors.

signals there

I 5 Chromatin

levels

factors

acetylated to

(TBP,

is formed,

adequate

transcription

times

specific

this

but

transcription

There to

complex

moderate

to

nucleosome

factors

Figure

Once

binds

the

tails

base

I 5 2. Figure

Promoter

pairs

Enhancers I 5 2. Enhancers

and

and

Upstream Upstream

Promoter Promoter

Elements

Elements

CHAPTER

Upstream Only

Promoter the

of

it

|

REGULATION

OF

EUKARYOTIC

from

the

upstream

an

promoter

enhancer.

element

Upstream

to

promoter

the

–25

elements

CCAAT



GC

box

sequence

HY

HY

(around

–75)

that

binds

a

transcription

factor

NF

transcription

factor

SP

MY

1 LY

rich

sequence

that

binds

a

general

in

following

the

DNA

are

binding

sites

for

activator

proteins.

Yield

Enhancers MEDIUM

They

may

be

up

to

1,000

base

pairs

away

from

the

HIGH

NOTE

have YIELD

the

characteristics: LOW



LY

1

High

Enhancers Enhancers

YIELD

The

IgMEDIUM heavy

the

large

They

may

gene •

be

they

The

located

upstream,

downstream,

or

YIELD

chain YIELD locus

intron

regions

for

has

separating

an the

LOW YIELD the variable

enhancer

in

coding

domain

from

the

gene. coding



EXPRESSION

include: MY



GENE

Elements

proximity

distinguishes

5

within

an intron of FUNDAMENTALS

the

regions

for

the

constant

domains.

FUNDAMENTALS

control.

orientation

of

the

enhancer

sequence

with

respect

to

REINFORCEMENT the gene is

REINFORCEMENT

not

important. •

They

can

proteins •

They

may

bending Similar There

are

appear that

to

act

interact be

of

brought

the

sequences

that

fewer

examples

in with

a

tissue

close

DNA

specific

them

are

to

the

manner

present

basal

only

if in

promoter

the

DNA

certain

region

binding tissues.

in

space

by

molecule.

bind

repressor of

these

proteins

in

sequences

Enhancer

elements

specific

eukaryotes

are

called

silencers.

known.

binding

transcription

NOTE

factors The

CRE

GRE

ERE

CREB

DNA

Estrogen

and as

E

Cortisol

binding

RNA pol 1

SP

in

the

sites

sequences response

elements,

of

that

vicinity for

(e.g.,

genes

proteins

are

often

serve called

regulators.

R

bending

NF

base

enhancers,

UPEs)

“cis” DNA

regulatory

promoters,

R

C

P

DNA

Transcription II

code

for

factors them)

are

(and

the

called

genes

that

“trans”

TFIID

1

regulators. TBP

diffuse

Trans through

regulatory the

cell

proteins to

their

point

can of

action. CAAT

UPE RICH

GC

TATA

Transcription

rich

Increased Promoter

elements

transcription

binding

factors

Figure Figure EnhancerEnhancer

and

general RNA

I I5 5 3.3. Stimulation Stimulation andand

Its Its

Associated Associated

(basal)

of

transcription

by

an

rate

polymerase

of Transcription of Transcription by

an

Transcription Transcription Factors

Factors

7

Immunology MY

PART

I

|

LY

BIOCHEMISTRY

Biochemistry Transcription The

High

Factors

activator

proteins

scription

factors.

domains,

that

Typically,

a DNA

bind

response

elements

transcription

binding

are

factors

domain

and

an

often

contain

activation

Yield

referred MEDIUM

at

least

2

to

H

as tran YIELD

ME

recognizable LOW

domain.

YIELD

Medical Genetics •

The

DNA

binding

promoter

or

motifs Behavioral Science/SocialSciences

response

have

families

domain element.

been

of

binds

to

specific

Several

characterized

transcription

a

types

and factors.

nucleotide

have

Some

of

been

sequence

DNA used

common

in

the

FUNDAMENTALS binding domain to

FUN

define certain REINFORCEMENT binding domains

DNA

RE

include: –

Zinc



Leucine



Helix

loop

helix



Helix

turn

helix

fingers

(steroid

zippers

homeobox •

The

Bind



Interact

dependent

transcription

(homeodomain

to

domain

other

proteins

allows

the

transcription

with

initiation –

(cAMP

receptors) factor)

encoded

by

homeotic/

genes)

activation



hormone

RNA

transcription

factors

polymerase

factor

and II

to

to:

coregulators

stabilize

the

formation

of

the

complex

Recruit

chromatin

modifying

proteins

such

as

histone

acetylases

or

deacetylases Two

types

can

scription

Table

I 5 1.

Important

be

Specific

Transcription

Factor

Response

(DNA

Protein)

(Binding

Steroid

cAMP

receptors

response

binding

element

(CREB)

Peroxisome

transcription

factors

and

specific

tran

Protein

Class

Element Site)

Function

HRE

Steroid

response

CRE

Response

Zinc

to

cAMP

finger

Leucine

protein

zipper

proliferator

activated

general

Factors

Transcription Binding

distinguished:

factors.

PPREs

Regulate

receptors

of

(PPARs)

multiple

lipid

aspects

Zinc

finger

metabolism

Activated

by

fibrates

and

thiazolidinediones

NFkB

(nuclear

kappa

factor

kB ——Regulates elements

expression

B)

many

genes

in

of

Rel

domains

immune

system

Homeodomain

proteins

Regulate

gene

during

General In

Transcription

eukaryotes,

RNA

polymerase

transcription. general

7

Helix

turn

helix

Factors

general II General

transcription

expression

development

transcription to

bind

and

transcription factor

TFIID

factors form

must

the

initiation

factors with

are its

TATA

bind

to

the

complex

promoter at

common

to

box

binding

most

to

the

start genes.

protein

allow

site The subunit

for

CHAPTER

5

|

REGULATION

OF

EUKARYOTIC

must

bind

examples

to

include

the

SP

TATA

1

and

box NF

before

1

that

RNA

polymerase

modulate

II

basal

can

bind.

transcription

Other MY

of

LY

Transcription

Specific

and

rate

of

silencer

factors

modulate of

sequences

time

in

Additionally,

hormones

factors.

Examples

of

regulatory

which factors

enhancer

(and

genes are

will

include

The in

contains exact a

cell

at

rates.

what can

activity

receptors

of

and

the

be

cell

YIELD to

or

specific

CREB

protein.

LOW BRIDGE

bind

proliferator

to

DNA

response

metabolism. vated

activated elements

Individual

by

a

variety

Fatty



Prostaglandin



Fibrates



Thiazolidinediones

natural

(PPARs)

(PPREs)

and

of

family

members of



receptors

and

this

xenobiotic

are

control of

caused genes

transcription

transcription

multiple zinc

ligands,

of

proteins

that

in

insulin

resistance

their

different

including:

seen

interaction

aspects

of

with

with

lipid

thiazolidinediones

PPARγ.

metabolism

is

Clofibrate

than

the

binds

thought

to

PPARα,

PEROXISOMES:

acute

year

(8.4

old pain

orders

HYPERTRIGLYCERIDEMIA

fasting

AND

for

and

the

8.9–10.3

mg/dL)

normal

1/100,

recessive

used

normal

producing would

Hardy

frequently

who

disease

prevalence

equation

A

the

simplification

2

FUNDAMENTALS NOTE

artificially

sweetened

professor producing

The

Her

her

births,

of

affected

carriers

The

carrier

the

the

risk

she

for

would

She

marry

a

of

from

and

asks

man

her genetics REINFORCEMENT

with

the

an

the known

is

much

the

REINFORCEMENT Hardy

disease

the

two

than

person

Weinberg

would

the

Equilibrium

estimate

in

the

PKU

is 1/10,000

live

the •

of

Allele

frequency

(1/10,000)

identifiable

to

of

births

First,

frequency

in

(PKU)

Prevalence

approximately

be

disease



is

reasons. the

equation of

population

higher,

for

Weinberg

prevalence

in

much

higher

affected

Hardy

PKU

a carrier

is

second,

of

carriers

marrying

condition

used

from

prevalence

frequency

this

geneticist

frequency

known

comes

homozygotes,

clinically.

(NutraSweetTM).

that

that

but

greatest

frequency

aspartame

chances

Phenylketonuria

tells live

1/50.

the

allele.

geneticist

1/10,000

with

about

the



Carrier

= =

1/100

frequency

=

=

0.01

2(1/100)

=

following 1/50

way:

Disease

prevalence

Carrier

frequency

q = 2q

square =

The

of

or

1/50,

marrying

now a

asks

carrier

q2

=

root

2/100,

woman

=

of

a

2q

=

1/10,000

(to

be

1/10,000, the

this

which

is

1/100

frequency

question:

allele,

births

calculated)

carrier

second

live

“Knowing

what

is the

that

probability

I have that

a

1/50

I will

have

chance

of

child

with

a

PKU?”

The

geneticist

answer

answers,

is based



The



If

“The the

probability he

is

normal

These

on

a

that

carrier,

allele

the to

probabilities

chance

joint

the

would

of

occurrence

she

will

of

marry

probability child

be

you

having two

a that

a

child

with

PKU

nonindependent

heterozygous he

will

carrier pass

is

1/100.”

This

events:

his

(1/50),

PKU

allele

and versus

the

(1/2).

multiplied

to

BRIDGE

TO

STATISTICS

If events

are

nonindependent,

probability

of

the

second

event,

has

occurred.

For

example,

1/50

×

1/2

=

1/100,

the

probability

event

husband

producing

allele

probability

that

by

the

assuming

what

student’s

is the will to

he

probability

that

the

probability pass

the

the

of

first

that

the

disease

the

child?

It is

will

be

a

the

probability

carrier

the (1/50,

give: event



one

multiply

that

she

will

have

a

child

with

PKU.

will (1/2,

1) pass event

multiplied the

by

disease 2),

assuming

causing

gene he

is

a

that

he

along carrier.

32

Biochemistry

PART

II

|

MEDICAL

GENETICS

Medical Genetics In

NOTE

summary,

there

equation Assuming

random

Weinberg

principle

relationship Behavioral Science/SocialSciences

frequencies

principle

can

autosomal

of

be

the

specifies

between

genotype

frequency

mating,

in applied

and

populations.

heterozygous

recessive

predictable

frequencies

to

3

major

terms

autosomal

one

usually

recessive

works

with

in

the

Hardy

Weinberg

conditions:



q2,

the

disease

prevalence



2q,

the

carrier

frequency



q,

This

estimate carriers

mutation.

to

Hardy a

allele

are

applied

the

frequency

of

the

disease

causing

allele

the of

an

When to

answering

identify

which

questions

which term

This

you

exercise



are

Hardy

terms

asked

to

heterozygous

of

the

For

two

calculations,

in

the

stem

it

of

the

is

important

question

and

points:

principle

can

in

recessive

carriers

is

effect,

given

be

applied

to

when

we

populations

estimate

the

know

only

prevalence the

prevalence

disease.

autosomal

gous

Weinberg

been

important

carriers

recessive

Hardy

has calculate.

Weinberg

of

In

involving

these

demonstrates

The



of

the

diseases,

much

vast

higher

such than

majority

of

as

the

PKU,

the

prevalence

prevalence

recessive

of

genes

are

of

affected

heterozy

homozygotes.

hidden

in

the

heterozy

gotes.

Hardy

Weinberg

The

calculations will

the

gous

be

dominant

diseases

heterozygous.

assumption

affected

severe

that

p

individuals

symptoms.



for

for

individuals use

Equilibrium

1/500

people

and

are

at

~

1.)

Diseases

must

In The

who,

For

Dominant

acknowledge

this

case,

term

q2

although

the

that

prevalence

most is

represents

the

much

less

commonly

have

a form

of

2q.

of

the

(One

prevalence

of

seen,

affected

can

may

again

homozy have

more

example,

in

the

United

increased

States

risk

for

cardiovascular

can

calculate

LDL

receptor

disease

and

deficiency

myocardial

infarction. •

Taking

2q

million

live

have

=

1/500,

greatly

are

elevated

cardiovascular present

one

births

LDL

disease with

that

homozygous

for

cholesterol

than

q2

the levels,

and

xanthomas,

1/106,

or

one

in

These

a much

heterozygotes,

characteristic

=

condition.

higher

are

more

xanthelasmas,

a

individuals risk likely

and

for to

corneal

arcus.

In

contrast,

repeats

in

gous

or

Sex

Chromosomes

When

homozygous

occur

but,

the

statement

the

disease

more

(autosomal

strongly

dominant),

with

disease

the

severity

number than

does

of

triplet

heterozy

status.

and

in

frequency

disease

much

considering

cases

32

Huntington

correlates

X

Allele

linked

Frequencies

recessive

hemizygous

conditions,

males

paradoxically,

it

“1/10,000

males

producing

allele:

(xY). also has

one

Therefore,

equals

q

the

must =

prevalence

hemophilia

A”

acknowledge

disease of

also

gives

affected the

1/10,000.



q2

=

prevalence

of

disease

in

females



2q

=

prevalence

of

female

carriers

(1/108, (1/5,000)

or

that

producing

1/100,000,000)

allele

most

allele males. frequency

Thus, for

CHAPTER

This

exercise



demonstrates

As

with

are



X

recessive

in

number

of

linked

|

POPULATION

GENETICS

that:

autosomal

hidden

2

female

these

traits,

the

heterozygous

genes

recessive

are

seen

traits

majority

carriers

are

in

affected

seen

of

X

linked

(although

recessive

genes

a considerable

males).

much

more

commonly

in

males

than

in

females.

FACTORS

RESPONSIBLE

IN/AMONG human loci,

populations

deviations

introduction

a new

mating

genetic

drift,

mutation

into

flow.

one

in can

example,

gene

than

typically

equilibrium

(for

and

more

are

from

of

nonrandom

often

GENETIC

VARIATION

POPULATIONS

Although most

FOR

Weinberg

produced

a population

contributes

factors

to

allele

new

of

effect),

natural

discussed

frequencies

the

(founder

action

are

for

mutations,

outside

the

these

equilibrium

by

from

consanguinity),

Although

effect

Hardy

be

selection,

independently,

in

a population.

NOTE Mutation Mutation,

discussed

in

populations.

to

population.

In

some

In

cases,

carrying

the

referred

to

general,

a new

as

a

drift

is

founder frequency

(see

below),

chain

disease) PA,

while

live

births.

chain

and

be

acts

upon

into of

a

be

genetic

much

expands

affected

someone

This

natural

DEHYDROGENASE

at

in to

this a

(maple

Mennonite

large,

the

a single

selection,

disease

a

variation

in



Mutation



Natural

selection



Genetic

drift



Gene

populations

responsible

for

are:

by flow

urine of

occurs (allele)

suggests

founder

syrup

community

mutation

group

factors

DEFICIENCY

deficiency the

evolutionary

is

through

by

4

genetic

population

when

community.

The

variation

from

population

the

rapidly can

in

of

due

very

new

in in

common

only the

Lancaster, 1/180,000

branched

origin

of

the

effect.

Selection

Natural

selection promote

frequencies disease

of

to do

or

fertility

alleles

that

reduce

alleles diseases,

the

recessive

genetic

survival

producing Dominant

exposed than

may

differ

community

births

population

gene

not

all

consanguinity.

predominance

This

the

KETOACID

live

of

founders

dehydrogenase

1/176

source

introduced

mutation

by

the do

early

the

CHAIN

U.S.

be

the As

of

dehydrogenase

Natural

rare.

the The

mutation.

that

in

in

of

ketoacid

occurs

rates

can

one effect.

the

ultimately

mutation

BRANCHED

Branched

is

mutation

mutation

generations, genetic

previously,

effects

helps in

of

diseases,

which

natural where

variation,

increasing

(referred

to

fitness. explain the

why

most

and

genetic

to

is typically

of

of

lower hidden

is

alleles

the

most

diseases allele

have

frequencies decreasing

fitness

causing tend

allele

fitness)

reduced

disease

selection, the

as

The

the

are more allele in

relatively readily

frequencies heterozygotes.

32

Biochemistry

PART

II

|

MEDICAL

GENETICS

Medical Genetics SICKLE

Sickle in

cell

some

disease parts

affects of

CELL

1/600

Africa.

DISEASE

African

How

AND

MALARIA

Americans

could

this

and

highly

up

to

deleterious

1/50

individuals

disease

causing

Behavioral Science/SocialSciences mutation that

become

the

Africa,

does

These

so

falciparum not

survive

individuals,

thus

high

is now

diseases

that

Cystic



Hemochromatosis



Glucose

frequency

in

for

33

of

sickle

some

of

(heterozygote

cell cell

and

in

the

much

heterozygotes.

disease,

are there

it

fact of

is

maintains

a

populations.

some

for

several

populations.

to

other

recessive

Examples

typhoid

advantage

dehydrogenase to

sickle

mutation,

resistance

resistance

lies in

Consequently,

advantages in

answer

common

sickle

malaria.

African

common

phosphate

The been of

cell

(heterozygote 6

has

signs

effects the

Africa?

in

include:

fever) iron

deficiency,

poor

environments)

hemolytic

anemia

malaria)

Drift

Mutation

small

clinical

heterozygote

relatively

(heterozygote

they

erythrocytes

the

no

for

fibrosis

in

lethal

advantage

evidence are



Genetic

the

in which

have

against

a heterozygote

There

especially parasite,

well

who

protected

relatively

frequent, malaria

rates can

result

do in

not

vary

significantly

significant

populations

or

founder

effects

common

(or

rarer)

act

Consider

the

pedigrees

are

along in

from

differences introduced with

small, (very

in by

genetic isolated small

a

population allele

founder

drift populations populations)

to

to

population,

frequencies effect.

make

Mutation

certain than shown

although

when

they rates

genetic

in

the in

occur

diseases

world Figure

at II

2

in

and

large. 1.

more

CHAPTER

Affected

person

either the

or

small

(founder

|

POPULATION

GENETICS

who

founds

into

I

2

moves

population

effect)

II

III

I

New

mutation

in

a

family

II

III

Genetic

drift

generation

begins. III

is

In 2/3,

both

II Figure 2 1.

Figure

than

the

woman

rather

than

would

be

6,

prevalence to

When

a

drift

alone A

would

many

natural the Hardy

the

or

can

the

in

a

top their

effect

allele

panel

more

in

1,000

drift

illustrates

less

children

(Generation

genetic

occurs or

had

offspring

example

founder

persons

Populations Populations

Although

this

affected

statistics.

Disease) Disease)

the in

mean. family,

mutation make

5)

disease

statistical

of by

Drift in Two Small Small in Two

(II

the

a single

frequency predicted

in

two

a

small

prevalent

than

III) affects points:

population, statistics

predict.

relatively

or

1/2

a Dominant with a Dominant

man of

than

new

with (Illustrated

affected

1/2,

larger

genetic



the

the

closer

populations



and

the

the

II 2 1. Genetic Genetic Drift

(Illustrated

If

examples

higher

large alleles disaster

population. Weinberg

population can

be

or

large

Genetic equilibrium

in

Hardy

affected scale drift

by

genocide may is

Weinberg

population

then

equilibrium “bottlenecks”

dramatically change

reduces allele

for in

frequencies

an

allele

which the

size

of

and

a

new

reached.

33

Biochemistry

PART

II

|

MEDICAL

GENETICS

Medical Genetics Gene

Flow

Gene

flow

flow,

refers

to

populations

frequencies.

the

exchange

located

close

Gene

flow

of to

can

also

cell

disease

genes

one

among

populations.

another

cause

often

gene

Because

tend

to

frequencies

to

have

of

gene

similar

change

gene

through

time:

Behavioral Science/SocialSciences The of

frequency gene

flow

disease cell

mutation

matings

produce

offspring

diseases

because

sectors in

(see

text

are

affected

with

individuals

more

likely

to

who

and

Consanguinity a union

individuals

share

ancestors

are mutations.

more

liable

to

the

U.S.

in

African

Americans

population

the

that

heterozygote

disappeared

Health

to is

related for

in

do

not

malaria

the

for

has

because

carry

advantage

because

part

the

sickle

become

rare

in

the

Consequences

mating

of

considered

at

the

to

second

individuals

be cousin

a consanguineous

who

are

consanguineous level

union.

related

if or

Because

closer). of

to

it occurs Figure

their

one

another

between II

mutual

2

2

illustrates

descent

from

disease

causing

a

share common

causing

lower

addition,

has

Its

refers

(typically,

recessive

box)

of

is

America.

pedigree

disease

other mutation;

Consanguinity

Consanguineous

common

sickle

from

causing

North

NOTE

of

ancestors,

genes.



Siblings



First



Second

These III

relatives

are

more

likely

to

share

the

same

Statistically,

(II

and

cousins

numbers

are a

has

3

3

4) IV

to

1/8

4)

III

causing a

II

1 and

referred

and

or

and

(IV

disease

it

II

(III

cousins

1 carries

brother)

2

as

the

1/2

share 2)

allele,

chance

share

1/8

their

of

share

of a 1/2

individual

genes

of

coefficients is

genes.

their

1/32

there

that

of

their

(1/2 genes

relationship. chance

III

4

1/2

×

1/2).

(1/8

×

1/2

×

Thus, that

(his

×

if individual

individual

first

cousin)

1/2).

III

3

has

it.

the

health

(his

I

II

III

IV

Figure II 2

Figure

Consequently,

there

consanguineous consequences show present frequency (e.g.,

33

is

an

matings. of

that

the with

uncle/niece

2 2. A Pedigree A Pedigree

increased

genetic

genetic

of

of

of

disease

brother/sister

cousin as

are

increases

disease studies

first matings

the

matings).

the

offspring

examined

matings.

These

approximately of

in

in have

cousin are

offspring further

Consanguinity Consanguinity

genetic

empirical

particularly first

disease or

Illustrating Illustrating

risk

Dozens

consanguinity,

offspring a

of

II 2.

the

twice

unrelated

matings.

offspring

of

closer

of

studies as

likely

The unions

to

CHAPTER

Review

1.

|

POPULATION

GENETICS

Questions

A

population

ing

has

genotype

been

assayed

counts

have

for

been

a

4

allele

polymorphism,

On

the

alleles

basis

of

and

2?

1

A.

0.38,

0.28

B.

0.19,

0.14

C.

0.095,

D.

0.25,

0.25

E.

0.38,

0.20

Which

the

A.

Consanguinity

B.

Genotype

Natural Once many

E.

The tion gotes

follow

Count

1,1

4

1,3

8

1,4

3

2,3

5

2,4

9

3,3

4

3,4

6

4,4

11

these

genotype

counts,

following

best

characterizes

has

no

frequencies

reverse

D.

the

what

are

the

gene

frequencies

of

0.07

of

C.

and

obtained:

Genotype

2.

2

is

not

has

population

in

to of

be the

be

no

Hardy

Weinberg

estimated

effect

deviates

generations frequency can

can

on

Weinberg

equilibrium?

equilibrium.

from

allele

frequencies,

but

the

true.

selection a

effect

Hardy

estimated

return

to

heterozygous if

on from

Hardy

Weinberg

equilibrium. equilibrium,

it

takes

equilibrium. carriers

one

Weinberg

Hardy

knows

of the

an

autosomal

incidence

recessive of

affected

muta homozy

population.

33

Biochemistry

PART

II

|

MEDICAL

GENETICS

Medical Genetics 3.

In

a

first

genetic

counseling

cousins

and

spring.

Which

A.

of

Because

session,

that

they

the

the

a healthy

are

following

couple

couple

concerned best

shares

about

has

revealed

health

risks

characterizes

these

approximately

half

that for

they

their

are off

risks?

of

their

type

of

genes,

most

of

the

Behavioral Science/SocialSciences offspring B.

The

are

couple

has

dominant C.

The

couple

The

to

be

affected

with

some

genetic

disorder.

an

increased

risk

of

producing

a child

with

an

autosomal

an

increased

risk

of

producing

a child

with

an

autosomal

disease. has

recessive D.

likely

disease. couple

has

an

increased

risk

of

producing

a

child

with

Down

syn

drome. E.

4.

There

An

is no

African They

African

American

provides

B.

Genetic

C.

Increased

gene

D.

Increased

mutation

E.

Natural

fibrosis

1/25

B.

1/50

C.

2/2,500

D.

1/2,500

E.

(1/2,500)2

a

known

and

that

mutation

33

1%

B.

0.5%

C.

0.25%

D.

0.1%

E.

0.05%

in

this

the

offspring.

two

children

seems

other

to

U.S.

with

be

more

sickle

cell

common

populations.

in

Which

of

the the

explanation?

population

in

this

population

in

from

this

with and

(1/100) (1/200) (1/400) (1/1,000) (1/2,000)

his

mate

of

carrier

of

recessive high

for

somebody

among

from will

a population

heterozygous

of carriers

of

a

population?

Suppose the

a

mutation

causing

condition.

urinary

seizures.

homozygous

1/2,500

incidence

autosomal

including

he

is

heterozygous

involved?

A.

than

predicted

ranges

is

mates

disease

fibrosis

the

an

population

bility

produced

this

best

rate

mutation

manifestations

man

in

cystic is

hyperprolinemia, is variable

flow

of what

is

for

selection

incidence

man

the

risk

drift

A.

A

has

why

population

Consanguinity

Europeans,

6.

asked

A.

cystic

in

couple

have

factors

If the

increase

American

disease.

following

5.

known

Phenotypic

excretion

of

that

0.0025%

mutation

causing

the

general

produce

a

proline

who

neurologic

(1/40,000) this

population, child

expression to

what is

of

condition.

homozygous

the

If the is

the

proba for

the

CHAPTER

7.

The

incidence

1/3,000 this

males. X

linked

A.

1/3,000

B.

2/3,000

C.

(1/3,000)2

D.

1/6,000

E.

1/9,000

of

Duchenne

On

the

recessive

muscular basis

for

this

dystrophy figure,

what

in is

North the

America gene

frequency

is

2

|

POPULATION

GENETICS

about of

mutation?

33

Biochemistry

PART

II

|

MEDICAL

GENETICS

Medical Genetics Answers

1.

Answer:

B.

The

obtained

by

adding

multiplying

Behavioral Science/SocialSciences

denominator

by

numerator

is

2

heterozygotes 3

alleles.

Dividing

by are

with

2.

and

2,4

The

Hardy

of

of

the

the

ing

number

expectation

(i.e.,

random

in

choice

a specific

and

1.

For

9

allele

2,

those copies

of

the

estimation

Using

the

frequency to

and

estimate

gen

the

fre

population.

equilibrium

in

a

by

above

violation

the

of

increas

equilibrium

the

assumption

of

be

from

gene

estimated

from

frequencies

(choice

genotype

B),

frequencies

(as

genotype

from

selection

can

the

population

cause

(e.g.,

deviations

affected

from

equilibrium

C). one

generation

C.

the

they

child

general

mating

couple

are

recessive

a

random

shares

more

likely

disease

with

an

is

required

to

return

a

population

D).

Because

autosomal

common to

causing

autosomal

be

ancestors

mutations.

recessive

(i.e.,

heterozygous Thus,

disease

one

carriers

is

of

their

risk

elevated

set

of

the

of

above

same

produc that

of

the

population.

First

cousins

Because

share both

harbor

a

with

a

allele

approximately

members

dominant

consanguinity

itself

is

syndrome it

mutation

is

not

the

(choice

thus

studies cousin

would

E).

are

that is

genes,

healthy,

mutation

elevate

the only

typically

an not

indicate couples

their

not neither

(choice of

copy

of

(choice

one B).

probability one

1/2

the

is

In

A).

likely

to

addition,

producing

a

disease

child

causing

disease.

D) by

of

couple

because

cause

transmitted

Empirical

(choice

does

to

and

the

causing

disease

needed

When

of

1/8

disease

dominant

Down

first

of (choice

grandparents),

33

allele:

gene

population

estimated

also

natural

Answer:

tion

4

1,3

A).

equilibrium

ing

be

can

eliminating

Only

of

1.

the 5

used

Weinberg

the

results

can

frequencies

(choice

3.

Hardy in

allele

allele of

population.

be

the

the

the

con

of

permits the

then

in

affects

consanguinity

homozygotes),

to

A)

frequencies

gene

By

can

type: 1;

0.14.

between

frequency

homozygotes

for

copy

homozygotes in

The

heterozygotes

yield =

each allele

copies

0.19 a

locus).

mating).

Genotype but

of

1,4

genotypes

mutation

genotype

(choice

the

of

is and

the

of of

19

have

relationship

gene

the

14/100

affected

heterozygous

Consanguinity

2 of

at

alleles

obtain

that

recessive

equilibrium

frequency,

quency

of

the

and

frequency

frequency

which (50)

alleles

of

we

These

a

100,

8 copies

alleles;

a gene

incidence

Weinberg

otype

8

heterozygotes

for

frequency

number

together,

genotypes.

respectively,

E.

the

yields of

two

contribute

another

this

has

the

genotype

is

individuals

individual

these

classes

2,3

2,

Answer: of

100,

frequency

genotyped

counting

1,1

Adding

two

the

allele

by the

gene

of

each

contribute

tribute

there

(because

with

the

number

obtained

homozygotes

of

the

is

affected be

the

affected the

approximately

result

female,

risk

it

by of

of acts

mutation. a

dominant

consanguinity.

genetic

double

a new like

disease that

of

in the

the

general

offspring popula

CHAPTER

4.

Answer:

E.

The

populations resistant is

to

disease

elevated

incidence

population African to

Although

is

no

6.

A.

frequency

for

frequency

is

Answer: will

also

(q2)

is

in

One a

be

allele

(probability

ty

7.

of

Answer: male dence is

of

a direct

mutation

are

disease,

POPULATION

GENETICS

which

the

mutation

population.

of

this

autosomal

account

for

the

American

for

obtained

causing or

their

the If

(probability 1/2),

and

the

occurrence

copy X

estimate

gives

males of

linked of

man the

the

have disease

is

the

mate

must

also

1/100

only

×

The

a

single

causing disease

the

frequency

×

in

the

X

1/2

pass along

the

determine =

frequency events The the

mate mutant

mutant the

allele probabili

1/400.

chromosome,

the

portion

the

mate

carrier

along

mutation.

in

man’s

mutation.

recessive male

carrier

homozygotes

independent

to 1/2

the

affected

Three

pass

the gene

The

1/200.

must

a

(0.02).

that

for

of

get

this

strong.

1/25.

of

1/100.

in

very

root

1/50

or

3 probabilities

recessive gene

of

homozygous

1/100),

Multiplying

Because

is

the

elevated is

to

frequency q,

2q) be

is

square

probability

the

frequency,

to

D)

the

2q,

determine

child

because

selection

(q)

into

decrease,

populations.

homozygotes)

mutation

allele

to

be

B).

populations

disease

(choice

approximately

carrier. the

cell

taking

affected

other

African

not

(choice

expected

natural

by

of

would drift

from be

rate

evidence

(approximately

one an

highest

the

first

then

joint

A. has

sickle

2pq,

must

1/2).

their

the

genetic

C) would

some

heterozygous

carrier

(probability

African

consequently of

(choice this of

is

by

for

a

many mutation

for

African

and

in

disease

population

must

flow

frequency

the

C.

happen

cell

not

the

levels

is

the

answer

given

be

must

does

large

frequency

that

the

1/40,000,

the

the

contrast,

This

(i.e.,

gene

disease

evidence

incidence

is

population,

this

In

Answer:

sickle

incidence

it in

elevated

been

increase,

population.

cell

in

the

disease

in

sickle

advantage

frequency

but

population

has

of

There

selective

elevate

specific

elevated

the

develop

a

family,

this

American

than

not

is

could

experienced

there

frequency

do

is

of

|

general.

have

African

rather

5.

of

disease

its

a specific

American

expected

the

A)

in

in

but there

elevating

(choice

recessive

cell carriers

Thus,

carriers,

Consanguinity

sickle

infection

recessive.

heterozygous

The

of

heterozygous

malarial

autosomal

in

frequency

because

2

each Thus, of

affected the

inci

a population

population.

33

3#

Cytogenetics

LEARNING

OBJECTIVES



Interpret



Solve



Demonstrate



Solve

scenarios

about

problems

basic

concerning

and

numerical

understanding

problems

definitions

of

concerning

terminology

chromosome

structural

abnormalities

chromosome

advances

in

abnormalities

molecular

cytogenetics

OVERVIEW This

chapter

reviews

alterations

in

chromosomes

or

alterations ly

1

vast

of

in

150

the

loss

of

and

are

of

fetuses

of

with

pregnancy

by

Thus,

the

microscopically may

They

observable

involve may

the

also

known

are

cause

of

abnormalities

are

seen

in

and

they

are

chromosome

50%

of

seen

20%

structural

seen

in

of

approximate

losses

fetuses the

The

prenatally:

fetal

are

extra

disability.

lost

spontaneous in

of

of

intellectual are

abnormalities

presence

consist

abnormalities

leading

chromosome

pregnancy,

trimester.

caused

alterations

chromosomes.

abnormalities

trimester

are

Chromosome

births

Chromosome

second

that These

chromosomes.

live

majority

first

diseases

chromosomes.

lost

during

during

leading

the

known

cause

DEFINITIONS

AND

X chromosome

contains

~1,200

Y chromosome

contains

~50

genes

of

loss.

BASIC

NOTE

the

genes

TERMINOLOGY

Karyotype Chromosomes

are

when

they

scope

to

are create

haploid

in

of

autosome,

the

with

the

an

an

typical

of

Y

(Figure

each

II

type

of

It

is

is

diploid,

chromosome.

chromosomes

(X

metaphase

the 3

Y)

23

1).

pairs

In

II

are in

3

the

both

the

II copies

1A,

a

presentation 3

1B of

is

is

a

each

ordered

lower

micro

human

Figure

Figure

mitosis,

the

of

chromosome; shown).

showing

placed

of

under

Chromosomes and

stage

photographed of

chromosome

male. the

the

are

display

cell of

each

individual

sex

ordered

somatic

copy

X and

during They

a drawing

one

karyotype

size,

a

visualized

condensed.

represents (only

easily

a karyotype,

chromosomes karyogram

most

maximally

according

right

to

portion

of

the

karyotype.

Metaphase the of

chromosomes

centromere, dyes

to

but reveal

can accurate

characteristic

be

grouped

identification banding

according requires

to

size

staining

and with

to

the one

position of

a

of

variety

patterns.

33

Biochemistry

PART

II

|

MEDICAL

GENETICS

Medical Genetics Chromosome To

banding

visualize

applied

Behavioral Science/SocialSciences



G

chromosomes so

G

that

banding. some

binds

DNA.

stains II

that

3

1 have

be

be

been

are

protein)

pattern

of

in

and

light

accurately

used

and

a similar

stained

with

1

1

1

1

1

1

1

2

2

3

3 4

digested

stained

(G

with

bands)

a karyotype.

manner.

stains

are

The

with

trypsin

Giemsa,

regions

that

There

are

chromosomes

(to

a dye

that

allow several

other

depicted

in

Figure

Giemsa.

1

2 p

1

1

1

1

2

2

1

1

1 2

1

q 2 2

2 q 2

in

2

2

various

partially

then

dark

identified

1

2

unambiguously,

chromosomes

2

3 p

a

to can

a karyotype

evident.

associated

reveals

chromosomes

is

Mitotic

digest

banding

in

banding

3

2

1

1

1

1

1

1

1

2 3

2

2

2

4

3

1

2

3

p 1 1

1 1

1 1

q 2

2

2

3

4

5

1

3

6

1

7

1

1

2

2

2

9

10

11

1

1

1 1 2

2

1

1

1

1

1

1 1 2

13

14

Negative

15

or

Positive

'Q'

Variable

bands

pale and

16

17

staining 'G'

bands

18

'Q'

and

Negative

19

'G'

20

bands 'R'

Positive

21

22

'R'

Y

bands

bands

A

B

Figure (Karyogram)

34

12

2

1

1

8

II

3 1. Human Metaphase Chromosomes. Figure II 3 1. Human Metaphase Chromosomes. and (B) Photograph of Metaphase (Karyogram) and (B) Photograph of Metaphase

(A) Idealized Drawing (A) Idealized Drawing Chromosomes (Karyotype) Chromosomes (Karyotype)

X

CHAPTER

Chromosome the

abnormalities

banding

tions)

pattern,

only

must

be

to

mitotic

cell

karyotype q,

relative

position

and

mere

is

the of

it

the

Smaller

at

the

by

looking

instance,

abnormalities

3

II 3

The

at

larger

dele

(microdeletions) end

1.

of

the

chapter.

p

One

of

the

sister

entered

long

instance, and

chromatids mitosis

arm

of

the

when

the

chromosome

characteristics

is

described

q

p

arm

are

the

4).

arm.

chromosome

q arms have

the the

the

chromosomes.

p

roughly

and

is

the

the

equal

arms

are

far

little 13,

have

the

15,

acrocentric

toward

one

evident.

toward

genetic 14,

centro

length.

displaced q

centromere

contains

Only

of

1)

centromere

The

Chromosomes

the

translocations,

which

chromosomes

are

called

1 contains

some

standard

Common

Symbols

Used

Autosome

Y

Sex

(

p.

two

has

one

end.

In

information, 21,

and

most

22

are

chromosomes

of

the are

in

22

or

and and The

have

chromosomes,

of

labeled

chromosome

on

centromere

(for

middle.

Robertsonian

II

is

chromosomes

residing

tips

a

chromosomes

involved

(+)

visually (for

CYTOGENETICS

centromere.

example,

acrocentric

X,

identified

discussed

interphase

chromosomes

(for

these

1

be

differences

(metaphase).

arm

the

Acrocentric

Table

4 Mb.

through

short

near

end

Table

can

reveals

(FISH),

performed

Submetacentric



about ways

gone

Metacentric



cases

contains

has

analysis

labeled

The

other

chromosome

the



of

in

some

technique

|

nomenclature

because



this

a resolution

identified

Chromosome Each

in

but

3

)

will

be

in

this

chapter.

telomeres.

nomenclature

in

discussed

Karyotype

applied

to

chromosomes.

Nomenclature

number

chromosomes

When

placed

before

chromosome

p

Short

q

Long

t

Translocation

del

Deletion

is

arm

of

arm

the

of

an

extra

autosomal or

number,

indicates

that

missing

chromosome

the

chromosome

NOTE Euploid

HY

23

Cells

(multiple HY

MY

MY •

Haploid

(23

chromosomes):

LY NUMERICAL

CHROMOSOME



Diploid

(46

somatic Euploidy When (sperm

cell and

each

pair);

both

members

has

a multiple

egg

cells)

they

are of

of are

said each

23

chromosomes,

euploid to

pair,

be or

cells haploid. 46

that Most

it is have somatic

23

said

to

be

MEDIUM euploid.

chromosomes cells

are

YIELD Gametes

(one diploid,

member

LOW YIELD containing

chromosomes.

REINFORCEMENT

most

Triploid

YIELD (69

MEDIUM condition

chromosomes):

rare

lethal

YIELD

of •

Tetraploid LOW rare

FUNDAMENTALS

chromosomes):

cells

HIGH

Yield •

a

gametes

LY

ABNORMALITIES

High

of

chromosomes)

(92YIELD chromosomes):

lethal

very

condition

FUNDAMENTALS

REINFORCEMENT 34

Biochemistry

PART

II

|

MEDICAL

GENETICS

Medical Genetics Two

types

of

humans:

Behavioral Science/SocialSciences

euploid

Triploidy

refers

Triploidy,

which

sperm are

cells, lost

have

cells

triploidy

and

to

with

cells

that

usually

prenatally.

contain

occurs

is common

numbers

defects

3

as

at

conception,

the

heart

However,

multiple

abnormal

of

chromosomes

are

seen

of of

but

about

of

copies

a result

1 in and

each

the

the

10,000 central

chromosome

(69

fertilization

of

vast

majority

live

births

nervous

of is

an

total).

ovum

these

a triploid.

system,

by

These

and

they

babies

do

not HY

Tetraploidy

refers

lethal

cases

to

cells

condition

have

been

is

that

contain

much

4

rarer

than

copies

of

each

triploidy

chromosome

among

live

(92

births:

) of

a few LY

High

Aneuploidy,

total). MY

Only

described.

Aneuploidy

(

2

conceptions

survive.

This

in

tetraploidy.

a deviation a specific

from

the

chromosome.

euploid

Two

number,

major

represents

forms

of

the gain MEDIUM

aneuploidy

are

(+)YIELD or

Monosomy



Trisomy

(loss

of

(gain

of

a

a

loss

ME

observed: LOW



H

Yield

YIELD

chromosome)

chromosome)

FUNDAMENTALS

FUN

REINFORCEMENT Autosomal Two

generalizations



All



Only

Trisomy

are

autosomal 3

live

helpful:

monosomies

autosomal

are

trisomies

inconsistent

(trisomy

with 13,

18,

a live

and

21)

are

birth. consistent

with

a

birth. 21

(47,XY,+21



Most



Intellectual



Short



Hypotonia



Depressed



Congenital



Increased



Alzheimer

or

common

47,XX,+21);

autosomal

Down

Syndrome

trisomy

disability stature

gene

342

R

aneuploidy

nasal heart risk

Reduced



Risk

upslanting

defects of

acute

disease on



bridge,

in

palpebral

approximately

40%

lymphoblastic

by

chromosome

fifth

fissures,

or

of

sixth

decade

(amyloid

21)

with

Trisomy

18

(47,XY,+18



Clenched

fist



Inward



Congenital



Low



Intellectual



Very

or

with

turning,

set

poor

prognosis

age

47,XX,+18);

Edward

Syndrome

fingers bottom”

feet

defects

micrognathia disability

maternal

overlapping “rocker

heart ears,

increased

fold

cases

leukemia

fertility increases

epicanthal

(small

lower

jaw)

precursor

protein,

APP

CHAPTER

Trisomy

Sex

13

(47,XY,+13



Polydactyly



Cleft



Microphthalmia



Microcephaly,



Cardiac



Very

(extra

lip,

renal

severe

tions

are

helpful:



If a



If

one

more

NOTE

the

X

sex

in

one

each

cell.

sex

syndrome.

Klinefelter

Syndrome



Testicular



Infertility



Gynecomastia



Female



Low



Elevated



High



Majority

is present,

required the

for

is

and

aneuploidy.

phenotype

chromosome

chromosome

common

autosomal

present,

aneuploidies

tends

Some

to

generaliza

Trisomy of

is the

most

spontaneous

common

loss

of

genetic

cause

pregnancy.

survival. is male

(with

all

one

are

but

Turner

minor will

exceptions). become

syndrome

a Barr

and

(47,XXY)

atrophy

NOTE distribution

of

hair

Genetic

Mosaicism

Genetic

mosaicism

in

Turner

Syndrome

testosterone FSH

and

pitched

LH

or

monosomy

or

within

a

Turner

syndrome

consistent

with

45,X

life

arise

47,XXX,

46,XY)



Females

with



Short



Edema



Cystic

cells

of

as

a condition

different

chromosome

single

and

constitutions

individual.

Some

are

mosaics

for

45,X

and

one

other

cell

lineage

women

with

are

at

increased

risk

for

gonadal

that

in Turner early

somatic are

46,XX

syndrome

embryogenesis

cells

that

or

47,XXX.

is by

thought

are

to

mechanisms

(46,XX, that

45,X;46,XY

in

have

others

Mosaicism

others

is defined are

45,XO)

45,X of

there

genotypes

(45,X

are

in which

voice

Syndrome

50%

X

is relatively

does

chromosome

than

Klinefelter



chromosomes than

is

important

Only

disability

defects

Y chromosome

body



eyes)

consequences

least

two

toes)

prognosis

involving

less

CYTOGENETICS

Syndrome

aneuploidy

have

At

Patau

and

intellectual and

Aneuploidy

Turner

fingers

(small

poor



47,XX,+13);

|

palate

chromosome

The

or

3

are

not

completely

understood

blastoma.

stature of

wrists

hygroma

and

in

ankles

in

utero

resulting

aorta

or

newborn

in

excess

nuchal

skin

and

heart

defect

in

“webbed”

neck •

Primary



Coarctation



Infertility



Gonadal

amenorrhea of

the

other

congenital

some

cases

dysgenesis

343

Biochemistry

PART

II

|

MEDICAL

GENETICS

Medical Genetics Nondisjunction Germ

is

cells

the

all

for

simplicity.

In

the

will

other diploid

with

will

egg

one

would

a

or

sperm.

The

homologous occur

for

live

each

original

cell

pair

is

shown

in

pair

of

homologs

When

fertilization

The

other

that

2,

fail

to

the

in

21)

the

cell.

occurs, gametes

are

sister

monosomy

of

the

conception

no

21,

during

Two the

with

chromatids

segregate One

fertilization

a

monosomy

is

the

has

the

the

no

21.

The

of

occurs,

gamete

of

(disjoin).

normally.

One is

When

conceptions

meiosis

syndrome. that

chromosome

normally

21.

segregate

a conception

example,

copy

of

a condition

birth.

21)

21.

(for

(disjoin)

syndrome.

in

during

Down

pair

Down

result

chromosomes

haploid

events

chromosome

chromosome

chromosome

in

haploid

only

segregate

with

nondisjunction

with

the

homologous

for

incompatible

example,

one

21 21

other

same

homologs

a trisomy

chromosome

In

produce although

The

of all

are be

aneuploidies

cell.

1,

gametes

to

of

chromosomes,

nondisjunction

meiosis

cause

meiosis

for

figure

within

usual

undergo

is diploid

Behavioral Science/SocialSciences

the

The

gametes

conception

copy

a

chromosome

sister

of

chromosome

remaining

two

(for

chromatids is

diploid

will

be

of

a trisomy

21

all

for

and

gametes

21

will

are

result

normal

ones.

Some

important



points:

Nondisjunction Edward,



Patau,

usual

Turner,

Nondisjunction

is

cause

and

more

of

aneuploidies

Klinefelter

likely

to

including

Down,

syndromes.

occur

during

oogenesis

than

during

age.

Environ

spermatogenesis. •

Nondisjunction

is

mental

agents

more

(e.g.,

likely

with

radiation,

increasing

alcohol)

maternal

appear

to

have

no

measurable

influence. •

Nondisjunction

is

CLINICAL

Surveys

of

copy

of

with

meiosis

MATERNAL

trisomy

chromosome

show

is

contributed

the

18th

Down

syndrome



For



At

age

35

the

risk

increases

to

about



At

age

40

the

risk

increases

to

1/100.



At

age

≥45

they

of

is are

3

of

the

is

Then

recombination

the

and

a Marker

0.01

0.05

0.10

0.20

0.30

0.40

score

0.58

1.89

3.47

2.03

–0.44

–1.20

interpreting

LOD

scores,

the

following

rules

apply:

LOD

score

more

likely

>3.00

shows

statistical

evidence

of

linkage.

(It

is

1,000

times

recombination

above

recombination

the

row

of

frequencies. the

LOD

• The

score

>3

LOD

that

score

times

value

at

is •

frequency.

An

the

gene

and

the

marker

are

linked

at

that

distance

than

likely

score

for most

of

likely

no

of

LOD

results

Gene

on

by



Linked

markers

genetic

testing

genetic

testing

Linkage

10

and

the

of

nonlinkage.

marker

are

(It

unlinked

is

100

than

linked

4

and

1

the

shows

score

is

analysis

indeterminate.

in

only

one

recombination

gene

>3.00,

and

the

with

analysis

the

the

case

is

there

frequency the

can

be

marker

cM can

may

used

several

along

show distant identify

5).

In

less

of

is

convincing 0.10.

evi

Therefore,

a recombination

locus

of with

a

disease

family

1% the

suggestive need

recombination

heterogeneity.

be

linkage,

but

gathered.

functions:

causing pedigree

markers

gene).

of to

important

practice,

than from

be

would

location

Chapter

must

data disease

serves

approximate

(see